You are on page 1of 61

SAMPLE CLAT TEST CODE AC001

Time Allowed: 2 hours /120 minutes Maximum Marks: 150

INSTRUCTIONS

1. The duration of the test will be 2 hours.


2. There are 150 questions. Each correct answer fetches 1 mark, and a wrong answer incurs
-0.25 marks.
3. Immediately after the commencement of the examination, you should check that this test
booklet does not have any unprinted or torn or missing pages or items, etc. If so, get it
replaced by a complete test booklet.
4. Please note that its candidate’s responsibility to fill in the roll number carefully without
omission or discrepancy. Any omission or discrepancy will render the answer sheet liable for
rejection.
5. You have to enter your roll number on the test booklet in the box provided alongside. Do
not write anything else on the test booklet.

6. This test booklet contains 150 questions printed in english. Each question comprises four
options . Select the option which you want to mark in the Answer Sheet.
7. All questions carry equal marks
8. There is a negative marking of - 0.25 marks, i.e. for each question answered wrongly by the
candidate, one-fourth mark assigned to that question will be deducted from the total
marks.
9. If a candidate gives more than one answer, it will be treated as a wrong answer even if one
of the given answers happens to be correct and there will be the same negative marking as
mentioned in point 8.
10. If a question is left blank i.e. no answer is given by the candidate there will be no negative
marking for that question.

DO NOT OPEN THIS TEST BOOKLET UNTIL YOU ARE ASKED TO DO SO


ENGLISH LANGUAGE

Direction: Read the following passage and answer the questions correctly.
Many managers are influenced by dangerous myths about pay that lead to counter productive
decisions about how their companies compensate employees. One such myth is that labor rates, the
rate per hour paid to workers, are identical with labor costs, the money spent on labor in relation to
the productivity of the labor force. This myth leads to the assumption that a company can simply
lower its labor costs by cutting wages. But labor costs and labor rates are not in fact the same: one
company could pay its workers considerably more than another and yet have lower labor costs if
that company's productivity were higher due to the talent of its workforce, the efficiency of its work
processes, or other factors. The confusion of costs with rates persists partly because labor rates are a
convenient target for managers who want to make an impact on their company's budgets. Because
labor rates are highly visible, managers can easily compare their company's rates with those of
competitors. Furthermore, labor rates often appear to be a company's most malleable financial
variable: cutting wages appears an easier way to control costs than such options as reconfiguring
work processes or altering product design.

The myth that labor rates and labor costs are equivalent is supported by business journalists, who
frequently confound the two. For example, prominent business journals often remark on the
"high" cost of German labor, citing as evidence the average amount paid to German workers. The
myth is also perpetuated by the compensation-consulting industry, which has its own incentives to
keep such myths alive. First, although some of these consulting firms have recently broadened their
practices beyond the area of compensation, their mainstay continues to be advising companies on
changing their compensation practices. Suggesting that a company's performance can be improved
in some other way than by altering its pay system may be empirically correct but contrary to the
consultants' interests. Furthermore, changes to the compensation system may appear to be simpler
to implement than changes to other aspects of an organization, so managers are more likely to find
such advice from consultants palatable. Finally, to the extant that changes in compensation create
new problems, the consultants will continue to have work solving the problems that result from
their advice.

1. The passage suggests that the “myth” mentioned in line 5 persists partly because
A. managers find it easier to compare their companies’ labor rates with those of competitors than to
compare labor costs
B. managers tend to assume that labor rates affect their companies’ budgets less than they actually
do
C. managers tend to believe that labor rates can have an impact on the efficiency of their
companies’ work processes
D. the average amount paid to workers differs significantly from one country to another

2. The author of the passage mentions business journals (line 39) primarily in order to
A. demonstrate how a particular kind of evidence can be used to support two different conclusions
B. cast doubt on a particular view about the average amount paid to German workers
C. suggest that business journalists may have a vested interest in perpetuating a particular view
Website: www.acingclat.com
YouTube: https://www.youtube.com/channel/UCCxsyMbrGVvbYu6Ua2eTb0A
Contact No. : +91-9166693196
D. identify one source of support for a view common among business managers

3. It can be inferred from the passage that the author would be most likely to agree with which of
the following statements about compensation?
A. A company’s labor costs are not affected by the efficiency of its work processes.
B. High labor rates are not necessarily inconsistent with the goals of companies that want to reduce
costs
C. It is more difficult for managers to compare their companies’ labor rates with those of
competitors than to compare labor costs.
D. A company whose labor rates are high is unlikely to have lower labor costs than other
companies.

4. The author of the passage suggests which of the following about the advice that the consulting
firms discussed in the passage customarily give to companies attempting to control costs?
A. It often fails to bring about the intended changes in companies’ compensation systems.
B. It has highly influenced views that predominate in prominent business journals.
C. It tends to result in decreased labor rates but increased labor costs.
D. It leads to changes in companies’ compensation practices that are less visible than changes to
work processes would be.

5. According to the passage, which of the following is true about changes to a company's
compensation system?
A. They are often implemented in conjunction with a company's efforts to reconfigure its work
processes.
B. They have been advocated by prominent business journals as the most direct way for a company
to bring about changes in its labor costs.
C. They are more likely to result in an increase in labor costs than they are to bring about
competitive advantages for the company.
D. They may seem to managers to be relatively easy to implement compared with other kinds of
changes managers might consider.

6. The primary purpose of the passage is to


(A) describe a common practice used by managers to control labor costs
(B) examine the relation between labor costs and other costs incurred by businesses
(C) explain why labor rates are a more significant factor than labor costs for most businesses
(D) identify a common misperception held by managers and point out some of the reasons for its
persistence

Direction: Read the following passage and answer the questions correctly.
Many theories have been formulated to explain the role of grazers such as zooplankton in
controlling the amount of planktonic algae (phytoplankton) in lakes. The first theories of such
grazer control were merely based on observations of negative correlations between algal and
zooplankton numbers. A low number of algal cells in the presence of a high number of grazers
suggested, but did not prove, that the grazers had removed most of the algae. The converse
Website: www.acingclat.com
YouTube: https://www.youtube.com/channel/UCCxsyMbrGVvbYu6Ua2eTb0A
Contact No. : +91-9166693196
observation, of the absence of grazers in areas of high phytoplankton concentration, led Hardy to
propose his principle of animal exclusion, which hypothesized that phytoplankton produced a
repellent that excluded grazers from regions of high phytoplankton concentration. This was the
first suggestion of algal defenses against grazing.

Perhaps the fact that many of these first studies considered only algae of a size that could be
collected in a net (net phytoplankton), a practice that overlooked the smaller phytoplankton
(nannoplankton) that we now know grazers are most likely to feed on, led to a de-emphasis of the
role of grazers in subsequent research. Increasingly, as in the individual studies of Lund, Round,
and Reynolds, researchers began to stress the importance of environmental factors such as
temperature, light, and water movements in controlling algal numbers. These environmental
factors were amenable to field monitoring and to simulation in the laboratory. Grazing was believed
to have some effect on algal numbers, especially after phytoplankton growth rates declined at the
end of bloom periods, but grazing was considered a minor component of models that predicted
algal population dynamics.

The potential magnitude of grazing pressure on freshwater phytoplankton has only recently been
determined empirically. Studies by Hargrave and Geen estimated natural community grazing rates
by measuring feeding rates of individual zooplankton species in the laboratory and then computing
community grazing rates for field conditions using the known population density of grazers. The
high estimates of grazing pressure postulated by these researchers were not fully accepted, however,
until the grazing rates of zooplankton were determined directly in the field, by means of new
experimental techniques. Using a specially prepared feeding chamber, Haney was able to record
zooplankton grazing rates in natural field conditions. In the periods of peak zooplankton
abundance, that is, in the late spring and in the summer, Haney recorded maximum daily
community grazing rates, for nutrient-poor lakes and bog lakes, respectively, of 6.6 percent and 114
percent of daily phytoplankton production. Cladocerans had higher grazing rates than copepods,
usually accounting for 80 percent of the community grazing rate. These rates varied seasonally,
reaching the lowest point in the winter and early spring. Haney’s thorough research provides
convincing evidence that grazers can exert significant pressure on phytoplankton population.

7. The author most likely mentions Hardy’s principle of animal exclusion in order to
(A) give an example of one theory about the interaction of grazers and phytoplankton
(B) defend the first theory of algal defenses against grazing
(C) support the contention that phytoplankton numbers are controlled primarily by
environmental factors
(D) demonstrate the superiority of laboratory studies of zooplankton feeding rates to other kinds
of studies of such rates

8. It can be inferred from the passage that the “first theories” of grazer control mentioned would
have been more convincing if researchers had been able to
(A) observe high phytoplankton numbers under natural lake conditions
(B) discover negative correlations between algae and zooplankton numbers from their field research

Website: www.acingclat.com
YouTube: https://www.youtube.com/channel/UCCxsyMbrGVvbYu6Ua2eTb0A
Contact No. : +91-9166693196
(C) understand the central importance of environmental factors in controlling the growth rates of
phytoplankton
(D) make verifiable correlations of cause and effect between zooplankton and phytoplankton
numbers

9. Which of the following, if true, would call into question Hardy’s principle of animal exclusion?
(A) Zooplankton are not the only organisms that are affected by phytoplankton repellents.
(B) Zooplankton exclusion is unrelated to phytoplankton population density.
(C) Zooplankton population density is higher during some parts of the year than during others.
(D) Net phytoplankton are more likely to exclude zooplankton than are nannoplankton.

10. The author would be likely to agree with which of the following statements regarding the
pressure of grazers on phytoplankton numbers?
I. ​Grazing pressure can vary according to the individual type of zooplankton.
II.​Grazing pressure can be lower in nutrient-poor lakes than in bog lakes.
III.​Grazing tends to exert about the same pressure as does temperature.
(A) I only
(B) III only
(C) I and II only
(D) II and III only

11. ​The passage supplies information to indicate that Hargrave and Geen’s conclusion regarding
the grazing pressure exerted by zooplankton on phytoplankton numbers was most similar to the
conclusion regarding grazing pressure reached by which of the following researchers?
(A) Hardy (B) Lund (C) Round (D) Reynolds

12. It can be inferred from the passage that one way in which many of the early researchers on
grazer control could have improved their data would have been to
(A) emphasize the effects of temperature, rather than of light, on phytoplankton
(B) disregard nannoplankton in their analysis of phytoplankton numbers
(C) collect phytoplankton of all sizes before analyzing the extent of phytoplankton concentration
(D) recognize that phytoplankton other than net phytoplankton could be collected in a net

13. According to the passage, Hargrave and Geen did which of the following in their experiments?
(A) They compared the grazing rates of individual zooplankton species in the laboratory with the
natural grazing rates of these species.
(B) The hypothesized population density of grazers in natural habitats by using data concerning the
population density of grazers in the laboratory.
(C) They estimated the community grazing rates of zooplankton in the laboratory by using data
concerning the natural community grazing rates of zooplankton.
(D) They estimated the natural community grazing rates of zooplankton by using data concerning
the known population density of phytoplankton.

Website: www.acingclat.com
YouTube: https://www.youtube.com/channel/UCCxsyMbrGVvbYu6Ua2eTb0A
Contact No. : +91-9166693196
14. Which of the following is a true statement about the zooplankton numbers and zooplankton
grazing rates observed in Haney’s experiments?
(A) While zooplankton numbers began to decline in August, zooplankton grazing rates began to
increase.
(B) Although zooplankton numbers were high in May, grazing rates did not become high until
January.
(C) Both zooplankton numbers and grazing rates were higher in December than in November.
(D) Both zooplankton numbers and grazing rates were lower in March than in June.

Direction: Read the following passage and answer the questions correctly.
In a 1984 book, Claire C. Robertson argued that, before colonialism, age was a more important
indicator of status and authority than gender in Ghana and in Africa generally. British colonialism
imposed European-style male dominant notions upon more egalitarian local situations to the
detriment of women generally, and gender became a defining characteristic that weakened women’s
power and authority.

Subsequent research in Kenya convinced Robertson that she had overgeneralized about Africa.
Before colonialism, gender was more salient in central Kenya than it was in Ghana, although age
was still crucial in determining authority. In contrast with Ghana, where women had traded for
hundreds of years and achieved legal majority (not unrelated phenomena), the evidence regarding
central Kenya indicated that women were legal minors and were sometimes treated as male
property, as were European women at that time. Factors like strong patrilineality and patrilocality,
as well as women’s inferior land rights and lesser involvement in trade, made women more
dependent on men than was generally the case in Ghana. However, since age apparently remained
the overriding principle of social organization in central Kenya, some senior women had much
authority. Thus, Robertson revised her hypothesis somewhat, arguing that in determining
authority in precolonial Africa age was a primary principle that superseded gender to varying
degrees depending on the situation.

15. The primary purpose of the passage is to


(A) present evidence undermining a certain hypothesis
(B) describe a particular position and its subsequent modification
(C) discuss two contrasting viewpoints regarding a particular issue
(D) describe how a social phenomenon varied by region

16. The passage indicates that Robertson’s research in Kenya caused her to change her mind
regarding which of the following?
(A) Whether age was the prevailing principle of social organization in Kenya before colonialism
(B) Whether gender was the primary determinant of social authority in Africa generally before
colonialism
(C) Whether it was only after colonialism that gender became a significant determinant of
authority in Kenyan society
(D) Whether age was a crucial factor determining authority in Africa after colonialism

Website: www.acingclat.com
YouTube: https://www.youtube.com/channel/UCCxsyMbrGVvbYu6Ua2eTb0A
Contact No. : +91-9166693196
17. The passage suggests that after conducting the research mentioned in highlighted text, but not
before, Robertson would have agreed with which of the following about women’s status and
authority in Ghana?
(A) Greater land rights and greater involvement in trade made women in precolonial Ghana less
dependent on men than were European women at that time.
(B) Colonialism had a greater impact on the status and authority of Ghanaian women than on
Kenyan women.
(C) Colonialism had less of an impact on the status and authority of Ghanaian women that it had
on the status and authority of other African women.
(D) The relative independence of Ghanaian women prior to colonialism was unique in Africa.

18. The author of the passage mentions the status of age as a principle of social organization in
precolonial central Kenya in highlighted text most likely in order to
(A) indicate that women’s dependence on men in precolonial Kenya was not absolute
(B) contrast the situation of senior women to that of less senior women in precolonial Kenyan
society
(C) differentiate between the status and authority of precolonial Kenyan women and that of
precolonial Ghanaian women
(D) identify a factor that led Robertson to revise her hypothesis about precolonial Africa

Direction: Read the following passage and answer the questions correctly.
Although numbers of animals in a given region may fluctuate from year to year, the fluctuations
are often temporary and, over long periods, trivial. Scientists have advanced three theories of
population control to account for this relative constancy. The first theory attributes a relatively
constant population to periodic climatic catastrophes that decimate populations with such
frequency as to prevent them from exceeding some particular limit. In the case of small organisms
with short life cycles, climatic changes need not be catastrophic: normal seasonal changes in
photoperiod (daily amount of sunlight), for example, can govern population growth. This
theory—the density-independent view—asserts that climatic factors exert the same regulatory
effect on population regardless of the number of individuals in a region.

A second theory argues that population growth is primarily density-dependent—that is, the rate of
growth of a population in a region decreases as the number of animals increases. The mechanisms
that manage regulation may vary. For example, as numbers increase, the food supply would
probably diminish, which would increase mortality. In addition, as Lotka and Volterra have shown,
predators can find prey more easily in high-density populations. Other regulators include
physiological control mechanisms: for example, Christian and Davis have demonstrated how the
crowding that results from a rise in numbers may bring about hormonal changes in the pituitary
and adrenal glands that in turn may regulate population by lowering sexual activity and inhibiting
sexual maturation. There is evidence that these effects may persist for three generations in the
absence of the original provocation. One challenge for density-dependent theorists is to develop
models that would allow the precise prediction of the effects of crowding.

Website: www.acingclat.com
YouTube: https://www.youtube.com/channel/UCCxsyMbrGVvbYu6Ua2eTb0A
Contact No. : +91-9166693196
A third theory, proposed by Wynne-Edwards and termed “epideictic,” argues that organisms have
evolved a “code” in the form of social or epideictic behavior displays, such as winter-roosting
aggregations or group vocalizing; such codes provide organisms with information on population
size in a region so that they can, if necessary, exercise reproductive restraint. However,
Wynne-Edwards’ theory, linking animal social behavior and population control, has been
challenged, with some justification, by several studies.

19. The primary purpose of the passage is to


(A) argue against those scientists who maintain that animal populations tend to fluctuate
(B) compare and contrast the density-dependent and epideictic theories of population control
(C) provide example of some of the ways in which animals exercise reproductive restraint to control
their own numbers
(D) summarize a number of scientific theories that attempt to explain why animal populations do
not exceed certain limits

20. It can be inferred from the passage that proponents of the density-dependent theory of
population control have not yet been able to
(A) use their theory to explain the population growth of organisms with short life cycles
(B) reproduce the results of the study of Christian and Davis
(C) explain adequately why the numbers of a population can increase as the population’s rate of
growth decreases
(D) make sufficiently accurate predictions about the effects of crowding

21. Which of the following, if true, would best support the density-dependent theory of
population control as it is described in the passage?
(A) As the number of foxes in Minnesota decrease, the growth rate of this population of foxes
begins to increase.
(B) As the number of woodpeckers in Vermont decreases, the growth rate of this population of
woodpeckers also begins to decrease.
(C) As the number of prairie dogs in Oklahoma increases, the growth rate of this population of
prairie dogs also begins to increase.
(D) After the number of beavers in Tennessee decreases, the number of predators of these beavers
begins to increase.

22. According to the Wynne-Edwards theory as it is described in the passage, epideictic behavior
displays serve the function of
(A) determining roosting aggregations
(B) locating food
(C) attracting predators
Website: www.acingclat.com
YouTube: https://www.youtube.com/channel/UCCxsyMbrGVvbYu6Ua2eTb0A
Contact No. : +91-9166693196
(D) regulating sexual activity

23. The challenge posed to the Wynne-Edwards-theory by several studies is regarded by the author
with:
(A) complete indifference
(B) qualified acceptance
(C) skeptical amusement
(D) perplexed astonishment

24. Which of the following statements would provide the most of logical continuation of the final
paragraph of the passage?
(A) Thus Wynne-Edwards’ theory raises serious questions about the constancy of animal
population in a region.
(B) Because Wynne-Edwards’ theory is able to explain more kinds of animal behavior than is the
density dependent theory, epideictic explanations of population regulation are now widely
accepted.
(C) The results of one study, for instance, have suggested that group vocalizing is more often used
to defend territory than to provide information about population density.
(D) Some of these studies have, in fact, worked out a systematic and complex code of social
behavior that can regulate population size.

Direction: Read the following passage and answer the questions correctly.
A mysterious phenomenon is the ability of over-water migrants to travel on course. Birds, bees, and
other species can keep track of time without any sensory cues from the outside world, and such
"biological clocks" clearly con-tribute to their "compass sense." For example, they can use the
position of the Sun or stars, along with the time of day, to find north. But compass sense alone
cannot explain how birds navigate the ocean: after a flock travelling east is blown far south by a
storm, it will assume the proper north-easterly course to compensate. Perhaps, some scientists
thought, migrants determine their geographic position on Earth by celestial navigation, almost as
human navigators use stars and planets, but this would demand of the animals a fantastic map
sense. Researchers now know that some species have a magnetic sense, which might allow migrants
to determine their geographic location by detecting variations in the strength of the Earth's
magnetic field.

25. The main idea of the passage is that


(A) migration over land requires a simpler explanation than migration over water does
(B) the means by which animals migrate over water are complex and only partly understood
(C) the ability of migrant animals to keep track of time is related to their magnetic sense
(D) knowledge of geographic location is essential to migrants with little or no compass sense

Website: www.acingclat.com
YouTube: https://www.youtube.com/channel/UCCxsyMbrGVvbYu6Ua2eTb0A
Contact No. : +91-9166693196
26. It can be inferred from the passage that if the flock of birds described were navigating by
compass sense alone, they would, after the storm, fly
(A) east
(B) north
(C) northwest
(D) south

27. In maintaining that migrating animals would need "a fantastic map sense" to determine their
geographic position by celestial navigation, the author intends to express
(A) admiration for the ability of the migrants
(B) scepticism about celestial navigation as an explanation
(C) certainty that the phenomenon of migration will remain mysterious
(D) interest in a new method of accounting for over-water migration

Direction: Read the following passage and answer the questions correctly.
A more comprehensive definition of "sea level," one useful to cartographers, requires a broad
understanding of various temporal and physical factors. First, consider the history of sea level: the
position at which sea and land meet, averaged over tidal stages and coastlines, has been remarkably
stable over time. Current sea level measures, though having risen very slightly over the last century,
are approximately the same as they were 35,000 years ago. Thus, the referent for terrestrial altitudes
and oceanic depths has been largely consistent for millennia, though cartographers' ability to make
accurate measurements has, of course, improved with advances in technology. Through the
processes of imaginative visualization and computer generation, much knowledge has been gained
regarding the predictable nature of sea level.

The oceans are governed primarily by Earth's gravity, which is strongest at the poles because of
Earth's slightly ellipsoid shape, the result of billions of years of rotation. What keeps the oceans
from pooling at the poles, drawn there by the greater gravitational forces, is the centrifugal force
generated by Earth's rotation. This force is strong enough to cause a bulge of ocean about eight
kilometers high around Earth's equator. Using sophisticated computer models, today's
cartographers can postulate how the intersection of sea and land would change should the Earth
ever cease to rotate on its axis. Over time, the bulge of ocean at the equator, no longer held in place
by the centrifugal forces, would flatten out. Drawn by the pull of the two poles, Earth's main
bodies of water would eventually become circumpolar, gathered in two massive pools around the
top and bottom, so to speak, of the globe, leading to the emergence of a supercontinent around the
planet's equator.

28. .According to the passage, which is true of the physical forces governing sea level?
A. Gravity, along with rotation, is responsible for Earth's slightly ellipsoid shape.
B. Centrifugal deterioration will eventually give rise to an enormous supercontinent.
C. The centrifugal force is the dominant influence on the shape of Earth's oceans.
D. The equatorial bulge is primarily a result of gravitational pooling at Earth's poles.

Website: www.acingclat.com
YouTube: https://www.youtube.com/channel/UCCxsyMbrGVvbYu6Ua2eTb0A
Contact No. : +91-9166693196
29. The author makes all of the following statements about historic sea levels EXCEPT
A. Variations in them have been noted in the last 35,000 years.
B. Their consistency has allowed cartographers to improve their measurements.
C. While mutable, they have, broadly speaking, maintained stability for millennia.
D. They are measured at many times and places to obtain a useful average.

30. The passage is chiefly concerned with


A. imparting an understanding of the effect of Earth's physical forces on sea life
B. arguing that the long-term stability of sea level assists in cartographic accuracy
C. attempting to compose an explanation of one of Earth's topographical variables
D. integrating hard data and thought experimentation to better visualize sea level

CURRENT AFFAIRS INCLUDING GENERAL KNOWLEDGE


Direction: Read the following passage and answer the questions correctly.
I. Finance Minister Nirmala Sitharaman announced some fresh relief measures for the economy,
the first such package after the second COVID-19 wave, focusing largely on extending loan
guarantees and concessional credit for pandemic-hit sectors and investments to ramp up healthcare
capacities. The government pegged the total financial implications of the package, which included
the reiteration of some steps that were already announced such as the provision of food grains to
the poor till November and higher fertiliser subsidies, at ₹[1].

Finance Minister Nirmala Sitharaman announced Rs [2] of additional credit for small and
medium businesses, more funds for the healthcare sector, loans to tourism agencies and guides, and
waiver of visa fee for foreign tourists as part of a package to support the pandemic-hit economy.
Together with previously announced Rs 93,869 crore spending on providing free foodgrains to the
poor till November and additional Rs 14,775 crore fertiliser subsidy, the stimulus package – mostly
made up of government guarantee to banks and microfinance institutions for loans they extend to
Covid-hit sectors – totalled up to Rs 6.29 lakh crore. She provided Rs 23,220 crore of additional
funding for setting up children and paediatric care/paediatric beds at hospitals to prepare
healthcare infrastructure to deal with any emergency arising for Covid wave hitting children. To
incentivize job creation, the government committed to paying the employer and employee’s share
to provident fund (PF) for all new recruitments done till March 2022. Previously, the government
paid Rs 902 crore for 21.42 lakh beneficiaries of 79,577 establishments.
Source: The Print ,
https://theprint.in/economy/nirmala-sitharaman-announces-new-covid-recovery-package-expands-
credit-relief/686108/

31. What is the total amount of economic relief released by the governed as replaced by [1] in the
above passage?
A. 6.5 lakh crore
B. 1.5 lakh crore
Website: www.acingclat.com
YouTube: https://www.youtube.com/channel/UCCxsyMbrGVvbYu6Ua2eTb0A
Contact No. : +91-9166693196
C. 6.28 lakh crore
D. 5.28 lakh crore

32. The relief package aims to prepare the health systems for emergency response and provide
impetus for growth and employment. However it will increase the Fiscal Deficit by _______as per
a report of State Bank of India. Fill in the blank.
A. 0.6%
B. 1.4%
C. 0.8%
D. 2.1%

33. Which of the following sectors is not covered in the economic relief package?
A. Insurance for higher education
B. Financial aid to power distributors
C. Extra fertiliser subsidy
D. Village Broadband

34. How much relief is decided to be given for expansion of the existing Emergency Credit Line
Guarantee Scheme as replaced by [2]?
A. 1.5 lakh crore
B. 1.1 lakh crore
C. 2.13 lakh crore
D. 50,000 crore

35. What is the target of the relief package for the medical sector?
A. Scaling medical infrastructure targeting non-metropolitan areas
B. Rapid building of medical pharmaceuticals, equipments in the rural areas
C. Providing medical insurance of upto INR 5 lakh to all the economically weaker people
D. All of the above

36. Which of the following is correct?


I. Atmanirbhar Bharat Rozgar Yojana incentivises employers for creation of new
employment, restoration of loss of employment through Employees’ Provident Fund
Organisation (EPFO).
II. Pradhan Mantri Garib Kalyan Ann Yojana: To the beneficiaries under the National Food
Security Act (NFSA), 2013, 5 kg of food grains per month free will be provided during
May-November 2021.
A. Only I
B. Only II
C. Both I and II
D. Neither I nor II

37. The free tourist visa will be given under the relief package for initial how many people for free
one month?

Website: www.acingclat.com
YouTube: https://www.youtube.com/channel/UCCxsyMbrGVvbYu6Ua2eTb0A
Contact No. : +91-9166693196
A. 5 lakh
B. 10 lakh
C. 3 lakh
D. 7 lakh

38. Extension of PLI scheme is given in which sector?


A. Telecom & Communication
B. Electronics Manufacturing
C. IT Software Manufacturing
D. Hardwares Manufacturing

Direction: Read the following passage and answer the questions correctly.
II. Prime Minister Narendra Modi on Saturday chaired a meeting to discuss steps which can be
taken to support children who have lost their parents due to COVID-19, and announced a series of
measures, including ensuring free education for such children. "All children who have lost both
parents or surviving parents or legal guardian/adoptive parents due to COVID-19 will be
supported under 'PM-CARES for Children' scheme," a release from the Prime Minister's Office
(PMO) said.

While announcing these measures, the prime minister emphasised that children represent the
future of the country and the country will do everything possible to support and protect the
children so that they develop as strong citizens and have a bright future. PM-CARES will
contribute through a specially designed scheme to create a corpus of Rs [1] for each child when he
or she reaches 18 years of age. This corpus will be used to give a [2] from 18 years of age, for the
next five years to take care of their personal requirements during the period of higher education.
"On reaching the age of 23 years, [3] the release said. Children up to 10 years of age will be given
admission in the nearest__________________. "If the child is admitted in a private school, the
fees as per the RTE norms will be given from the PM CARES," the release said, adding it will also
pay for expenditure on uniform, textbooks and notebooks.
Source: Business Today
https://www.businesstoday.in/latest/economy-politics/story/govt-to-ensure-free-education-for-chil
dren-who-lost-parents-to-covid-19-297272-2021-05-29
39. What is the amount of corpus replaced by [1] in the passage?
A. 5 Lakh
B. 10 Lakh
C. 15 Lakh
D. 11 Lakh

40. What is the eligibility of applying under PM Cares for Children’s Scheme?
A. Children who have lost their both parents or grandparents
B. Children whose legal guardians have lost their jobs
C. Children whose parents have lost their jobs
Website: www.acingclat.com
YouTube: https://www.youtube.com/channel/UCCxsyMbrGVvbYu6Ua2eTb0A
Contact No. : +91-9166693196
D. All of the above

41. How is the corpus of [1] (in passage) going to be used for children as replaced by [2] & [3] in
the above passage?
A. Monthly stipend and on reaching the age of 23, will receive the lump sum amount for
personal use
B. One time transfer of 12 months stipend till the age of 23 and providing 5 lakh after
reaching 23 years
C. Monthly stipend of 5,000 to girls and 4,000 to boys till the age of 23 and then giving INR
5 Lakh after 23
D. None

42. What should come in the blank given in the passage for education expenses of younger
children?
A. Admission to Navodaya Vidyalaya or private school as a hosteller or day scholar
B. Admission to State Boards Schools or Kendriya Vidyalaya as hosteller or day scholar
C. Admission to Kendriya Vidyalaya, Navodaya Vidyalaya or Private School as day scholar
D. Admission to Kendriya Vidyalaya or in a private school as a day scholar
43. Which of the following is incorrect about the Health Insurance to children under the scheme
mentioned in the passage?
I. All children will be enrolled as a beneficiary under Ayushman Bharat Scheme with a health
insurance cover of Rs. 15 lakh.
II. The premium amount will be paid by PM-CARES till a child turns 18.
A. Only I
B. Only II
C. Both
D. None

44. PM-Cares fund is a public charitable trust under the chairmanship of ?


A. Prime Minister
B. Home Minister
C. Finance Minister
D. Health Minister

45. The NCPCR has devised an online tracking portal for children in need of care and protection.
This has been created with a purpose for online tracking and digital real time monitoring
mechanism of children who are in need of care and protection. What is the name of the online
portal mentioned in this question?
A. Bal Swaraj
B. Bal Swastha Suraksha
C. Bal Suvidha
Website: www.acingclat.com
YouTube: https://www.youtube.com/channel/UCCxsyMbrGVvbYu6Ua2eTb0A
Contact No. : +91-9166693196
D. Bal Samvedna

46. NCPCR mentioned in the above question is under administrative control of which ministry?
A. Ministry of Health and Family Welfare
B. Ministry of Education
C. Ministry of Women and Child Development
D. Ministry of Finance

Direction: Read the following passage and answer the questions correctly.
III. India on May 27 abstained on the [1]'s resolution to open an international investigation into
violations surrounding the latest Gaza violence, and into "systematic" abuses in the Palestinian
territories and Israel. The resolution was nevertheless passed with 24 of the council's 47 members
voting in favour of the probe. This is the council's first-ever open-ended commission of inquiry
(COI) -- the highest-level investigation that can be ordered by the council. Countries that voted in
favour include Pakistan, China, Bangladesh, Russia, Mexico and Sudan, while Germany, UK and
Austria were among those who voted against it. The independent investigation will have a broad
mandate to look into all alleged violations, not just in Gaza and the occupied West Bank, but also in
Israel during hostilities that were halted by a ceasefire on May 21.

The Palestinian foreign ministry saluted a move saying it "reflects the determination of the
international community to move forward in the path of accountability, law enforcement, and
protection of Palestinian human rights." The investigators, the text said, should probe "underlying
root causes of recurrent tensions and instability, including systematic discrimination and repression
based on group identity".
Source: MSN,
https://www.msn.com/en-in/money/topstories/israel-palestine-conflict-india-abstains-on-unhrc-re
solution-probe-ordered-after-24-members-vote-in-favour/ar-AAKsNnX

47. Which international organisation is replaced by [1] in the above passage?


A. UNDP
B. UN ECOSOC
C. UNHRC
D. UNSC

48. What is the geographical location of Jerusalem?


A. North East of Gaza
B. North of Golan Heights
C. West of Mediterranean sea
D. All of the above

49. Which of the following countries other than India did abstain from voting in the resolution?
A. France
B. Turkey

Website: www.acingclat.com
YouTube: https://www.youtube.com/channel/UCCxsyMbrGVvbYu6Ua2eTb0A
Contact No. : +91-9166693196
C. USA
D. North Korea

50. Which of the following is correct about India’s position till date on the Israel-Palestine issue?
A. India recognised Israel in 1950
B. India is also the second non-Arab country after USA, to recognise Palestine Liberation
Organisation (PLO) as the sole representative of the Palestinian.
C. India is also one of the first countries to recognise the statehood of Palestine in 1946
D. All of the above

51. Which of the following is correct about Israel?


I. Israel today is a small country in West Asia, about the size of one of India’s northeastern
states, Meghalaya or Manipur.
II. Israel is bordered by the Black Sea to its west, Lebanon to the south, Jordan and Syria to its
east and Lebanon to the north.
III. Israel can be traced back to the biblical figure Abraham, who is deemed the father of
Judaism (through his son Isaac) and a patriarch of Islam (through his son Ishmael).
A. I and II
B. II and III
C. I, II and III
D. I and III

52. Which declaration was made in 1917 by the British government, hoping to gain Jewish support
for World War I, which promised: “the establishment in Palestine as a national home for the Jewish
People”?
A. Balfour Declaration
B. Versailles Declaration
C. Solomon Declaration
D. Queen Victoria Declaration

53. Which of the following is false about the 6 day war of Israel & Hamas?
A. In 1967, the 6-Day War started in which Israel won control of the Gaza Strip, the West
Bank, the Golan Heights, and the Sinai Peninsula. Israel captured East Jerusalem also.
B. Hamas, a Shia Islamist militant group won the elections in Palestine in 2021
C. Hamas rules over Gaza. Gaza’s borders are tightly controlled by Israel and Egypt.
D. The West Bank is still occupied by Israel.

54. Which of the following is correct about the latest development of the 2021 Israel-Palestine
dispute?
I. Began when Palestinians protested against an anticipated decision of the Israeli Supreme
Court over the eviction of six Palestinian families from Sheikh Jarrah in occupied East
Jerusalem.
Website: www.acingclat.com
YouTube: https://www.youtube.com/channel/UCCxsyMbrGVvbYu6Ua2eTb0A
Contact No. : +91-9166693196
II. The next day, Israeli Police stormed the Al Aqsa mosque.
III. A few days later, Hamas and other Palestinian groups started firing rockets from Gaza into
Israel to which Israel retaliated.
A. I and II
B. None
C. I, II and III
D. II and III

55. Which of the following is true about the significance of Jerusalem for Israel And Palestine?
A. Israel claims the 60% of undivided city as its rightful capital
B. Palestinians state their right of freedom and self-determination.
C. The city is also considered a holy one having many places of religious importance to the
three Abrahamic religions of Judaism and Islam.
D. B and C

Direction: Read the following passage and answer the questions correctly.
IV. The Supreme Court recently directed states and Union Territories to implement the 'one
nation, one ration card' scheme by July 31, while asking the Centre to provide dry ration for free
distribution among migrant workers till the COVID-19 situation continues. A bench of justices
Ashok Bhushan and M R Shah issued a slew of directions on a plea of three activists seeking
directions to the Centre and states to ensure food security, cash transfers and other welfare
measures for migrant workers who faced distress again due to curfews and lockdowns in various
parts of country during the second wave of COVID-19.

The bench has said ONORC scheme is for welfare of poor citizens as it allows migrant workers to
get ration at the place of their work in other states as well where their ration cards are not registered.
The bench directed the Centre to develop a portal with the help of the National Informatics
Centre (NIC) for registration of unorganised sector workers for granting them the benefits of
welfare schemes by July 31. It also directed the states, UTs to run community kitchens for migrant
workers till the COVID pandemic continues. It asked the Centre to keep allocating foodgrains to
the states, UTs for distribution among migrant workers for free till the pandemic situation exists.
The state governments and the UT administrations shall bring a scheme to provide dry ration to
migrant workers by July 31 and such a scheme will remain till COVID situation continues, the top
court said.
A national platform digitises ration cards that have all relevant details of beneficiaries and schedules
a monthly quota of ration the card holder is entitled to receive. Aadhaar is the main biometric data
source. The system allows withdrawals of ration through electronic point of sale machines. In
effect, One Nation, One Ration Card allows portability of ration benefits across states and it works
just like mobile number portability.

Source: Business Today

Website: www.acingclat.com
YouTube: https://www.youtube.com/channel/UCCxsyMbrGVvbYu6Ua2eTb0A
Contact No. : +91-9166693196
https://www.businesstoday.in/latest/economy-politics/story/supreme-court-directs-states-uts-to-i
mplement-one-nation-one-ration-card-scheme-by-july-31-300021-2021-06-29

56. The ONORC is not exactly a 2020 product. It was launched in which year?
A. 2019
B. 2018
C. 2017
D. 2016

57. Which of the following is correct about the ONORC scheme for migrant workers?
I. The beneficiary is not required to be a registered ration card holder in the state, district or
village of work or stay.
II. Another provision is that if one family member (migrant worker) receives her quota of
ration at one place (say Delhi), the rest of the family members are still entitled to getting
ration in their village in other district (say Cachar in Assam).
A. Only I
B. Only II
C. Both
D. None

58. On which of the following supporting portals does the national portal runs as mentioned in the
passage?
I. Annapoorti
II. Annavitran
III. Integrated Management of Public Distribution System (IM- PDS)
IV. One Nation Public Distribution System (ON-PDS)
A. I and III
B. II and III
C. I and IV
D. II and IV

59. Which of the following states have adopted the One Nation One Ration Card Scheme?
A. Madhya Pradesh
B. Delhi
C. Chhattisgarh
D. Assam

60. Which of the following statements are correct about the NFSA, 2013?

Website: www.acingclat.com
YouTube: https://www.youtube.com/channel/UCCxsyMbrGVvbYu6Ua2eTb0A
Contact No. : +91-9166693196
I. The beneficiary AAY household is entitled to 50 Kg of food grains per month
II. Each PHH person is entitled to 10 Kg of foodgrains per month under the National Food
Security Act.
III. There has been no revision in the prices of the foodgrains in the Union Budget 2021. The
PDS prices of rice, wheat and coarse grains will continue to be sold at Rs. 3, Rs. 2 and Re. 1
per kg, respectively.
A. Only I
B. Only III
C. I and II
D. I, II and III

61. Which international goal is set to be achieved with implementation of NFSA 2013 in India?
A. FAO’s: Food for All
B. UNGA’s Sustainable Development Goal of Zero Hunger
C. UNHRC’s Nutrition for All
D. Both A and B

62. What are the objectives of ONORC Scheme?


I. Reforms in the public distribution system.
II. Access of food grains to each beneficiary.
III. Integrating all the Indian states and UTs in the scheme by March 2021.
A. I and II
B. II and III
C. Only I
D. I, II and III

63. Right to food is covered under which fundamental right?

A. Right to freedom of trade and business


B. Right to life
C. Right to equality
D. Right against discrimination

Direction: Read the following passage and answer the questions correctly.
V. As many as 60 scientists, including marine ecologists, marine biologists, wildlife biologists, coral
reef ecologists and experts from other fields have written to President Ram Nath Kovind,
requesting his intervention to withdraw the Lakshadweep Development Authority Regulation
2021. They said the regulation is in contravention of several existing laws. The regulation gives the
government powers to choose any plot of land for development, and can override any public
opposition. Lakshadweep administration Praful Khoda Patel’s proposal has triggered widespread
protests in the islands.In the letter, the Lakshadweep Research Collective said that after a thorough

Website: www.acingclat.com
YouTube: https://www.youtube.com/channel/UCCxsyMbrGVvbYu6Ua2eTb0A
Contact No. : +91-9166693196
review of the draft, they found it to be in contravention of several existing laws, including the Land
Acquisition, Rehabilitation and Resettlement Act, 2013, the Biological Diversity Act 2002, The
Environment (Protection) Act, 1986 as well as the recommendations of the [1] Committee set up
by the Supreme Court.
Lakshadweep is a coral atoll, which means the islands are part of a living coral system. “Anyone who
has lived or worked in Lakshadweep for any length of time will be aware of its special vulnerability...
Given how linked land, lagoon and reef are in Lakshadweep, the development envisioned in the
draft LDAR would be nothing short of disastrous,” said Rohan Arthur, Senior Scientist, Nature
Conservation Foundation.
Source: MSN,
https://www.msn.com/en-in/news/other/e2-80-98lakshadweep-development-draft-against-laws-th
at-safeguard-ecology-livelihood-culture-e2-80-99-60-scientists-write-to-president-seek-intervention
/ar-AALpzTX

64. Which committee’s recommendation is violated by the Regulation as replaced by [1] in the
passage?
A. Justice Raveendran Committee
B. Justice B N Srikrishna Committee
C. Justice A. M Khanwilkar Committee
D. Justice Lodha Committee

65. Which of the following is correct about the Lakshadweep Development Authority to be
constituted under the Regulation?
I. It empowers the government, identified as the administrator, to constitute Planning and
Development Authorities under it to plan the development of any area identified as having
“bad layout or obsolete development”.
II. The authority would be a body corporate with a government-appointed chairman, a town
planning officer and three ‘expert’ government nominees besides two local authority
representatives.
III. These authorities are to prepare land use maps, carry out zonation for type of land use and
indicate areas for proposed national highways, arterial roads, ring roads, major streets,
railways, tramways, airports, theatres, museums etc. z Only cantonment areas are exempted
from this.
A. I and II
B. I and III
C. I, II and III
D. Only III

66. Which of the following languages is spoken in Lakshadweep?


A. Malayalam and Mahi
Website: www.acingclat.com
YouTube: https://www.youtube.com/channel/UCCxsyMbrGVvbYu6Ua2eTb0A
Contact No. : +91-9166693196
B. Marathi and Malayalam
C. Urdu and Tamil
D. Kannada and Tamil

67. Which of the following is correct about the fees for changing zones under the constitution of
the Development Authority?
I. It stipulates that islanders must pay a processing fee for zone changes.
II. It implies that localities would be required to pay fees to gain approval to alter zones as per
the development plan, as well as fees for permission to develop their own land.
A. Only I
B. Only II
C. Both
D. None

68. Which of the following is correct about Lakshadweep?


I. India’s smallest Union Territory, Lakshadweep is an archipelago consisting of 36 islands
with an area of 32 sq km. €
II. It is directly under the control of the Kerala State Government
III. There are three main groups of islands: Amindivi Islands, Laccadive Islands and Minicoy
Island.
IV. Amindivi Islands are the southernmost while Minicoy island is the northernmost
A. I and III
B. II and IV
C. I, II and IV
D. II, III and IV

LEGAL REASONING
DIRECTIONS (Q69 - Q73): Read the paragraph and answerthe following:
At present the legal regime of trafficking in humans is explicitly and implicitly governed by the
following statutes towards curbing the menace. The table containing relevant provisions is as
follows:
Immoral Traffic (Prevention) Act, 1956
Section 3 – It provides for punishment to a person for keeping a brothel or allowing premises to be
used as a brothel or who is in charge of any such premises either by himself or through a tenant,
occupier, etc.
Section 4 – It provides for punishment to any person over 18 years of age, living on the earnings of
prostitution of another person.

Website: www.acingclat.com
YouTube: https://www.youtube.com/channel/UCCxsyMbrGVvbYu6Ua2eTb0A
Contact No. : +91-9166693196
Section 5 – It provides for punishment to any person who is involved in procuring, inducing or
taking another person for the sake of prostitution.
Section 6 – It provides for punishment to a person who detains another person with or without his
consent in any brothel or any premises for prostitution with an intent that such detained person
may have sexual intercourse with any person who is not the spouse of such detained person.
Section 7 – Any person who carries on prostitution and the person with whom such prostitution is
carried on in any premises which is within close proximity to a public place, including a hospital,
nursing home, place of religious worship, hostel, educational institution, or in an area notified
under the provisions of the Act, can be punished with imprisonment for a term of three months.
Section 18 – A Magistrate can order the immediate closure of a place that is being used as a brothel
or as a place for prostitution and is within 200 meters of any “public place” as referred to in Section
7 above, and direct the eviction from the premises from where any person is ostensibly carrying out
prostitution on receipt of information from the police or otherwise. The occupier is given only
seven days notice for eviction from such premises.

69. A owns a premise in XYZ colony. A allows B to run a brothel in the premises that he holds in
XYZ colony. As per the above passage, will A be punishable for the above stated act?
A. Yes, he will be punishable under section 5.
B. Yes, he will be punishable under section 3.
C. Yes, he will be punishable under section 4.
D. No, only B will be punished.

70. A and B are brother and sister respectively. A is younger to B and B is 17 years old. To survive
and support both of them, B decided to practice prostitution. Will A be liable as per the above
passage?
A. Yes, because A didn’t stop B from being a prostitute.
B. No, because prostitution is legal in India.
C. No, because he is a minor.
D. Cannot be determined.

71. A and B were best friends when they were kids. When they were 12 years old, B ran away to
mumbai in search of work. She started practicing prostitution. After three years, A also went to
Mumbai to earn and met B in a brothel. After looking at B’s lifestyle, A also decided to practice
prostitution and starts working in the same brothel as B. As per the above passage, which of the
following statements is true?
A. A will be punished under section 4.
B. B will be punished under section 4.
C. B will be punished under section 3.
D. None of the above.

72. A owns a premise in XYZ colony. The premise is used as a brothel. The brothel is situated 300
meters away from RST Law School and is in very close proximity. As per the above passage, in how
many days can it be closed down by the magistrate?
A. Immediately.

Website: www.acingclat.com
YouTube: https://www.youtube.com/channel/UCCxsyMbrGVvbYu6Ua2eTb0A
Contact No. : +91-9166693196
B. 7 days.
C. 15 days.
D. None of the above.

73. A was a drug addict and spent all his money on drugs. A’s friend advised him to ask B (A’s
sister) to give him money for buying more drugs. A as per his friend's advice, asks money from his
sister B, who practices prostitution for earning her livelihood. As per the above passage, under
which section will A be punished?
A. Section 3.
B. Section 4.
C. Section 5.
D. Data Inadequate.

74. A and B are brother and sister respectively. A is older to B and B is 21 years old. To survive and
support both of them, B decided to practice prostitution. Will A be liable as per the above passage?
A. Yes, because A didn’t stop B from being a prostitute.
B. Yes, because prostitution is illegal in India.
C. Yes, because he is a major.
D. Data Inadequate.

75. A owns a premise in XYZ colony. A allows B to run a brothel in the premises that he holds in
XYZ colony. The premise is not situated near any of the public places as mentioned in section 7. As
per the above passage, will A be punishable for the above stated act?
A. Yes, he will be punishable under section 5.
B. Yes, he will be punishable under section 3.
C. Yes, he will be punishable under section 4.
D. No, only B will be punished.

DIRECTIONS (Q76 - Q80): Read the paragraph and answerthe following:


A Will is a legal declaration of the intention of a person (testator) with respect to his property or
estate, which he desires to take effect after his death. Section 18(e) of the Indian Registration Act,
1908 specifies that the registration of a will is optional. The mere fact that registration of the will is
made optional, it cannot be said that because of its non-registration, an adverse inference can be
drawn.
A will although registered can be challenged in the court of law. The mere fact that a will has been
registered will not, by itself, be sufficient to dispel all suspicions regarding it. A registered will may
not be the last testament. A new will made, even if unregistered, if valid, will trump the registered
will. A will must be made by the testator and duly signed by him. Signature or thumb impression
of the testator is not the only requirement. Along with the testator, the will must contain the
signature or thumb impression of two witnesses, witnessing that the will belongs to the testator. If
any of these is not present, there is a lack of due execution in the will and the same can be
challenged in the court of law.
A will or any part of the will, the making of which has been caused by fraud, coercion, undue
influence is bad in the eyes of the law. It is well-settled law that once the execution of a will is

Website: www.acingclat.com
YouTube: https://www.youtube.com/channel/UCCxsyMbrGVvbYu6Ua2eTb0A
Contact No. : +91-9166693196
proved, the burden to prove that it was fabricated or manufactured or was obtained by committing
fraud, coercion or undue influence is upon the shoulder of the objector of such will.

76. A is the testator of an unregistered will. In his will, he has allotted 60% of his wealth to his son
and the rest 40% to an organisation, which works for the orphan children. After A’s death, his son
contends that since the will was unregistered, 40% of the wealth cannot be given to the mentioned
organisation. Whether the son is right? Decide as per the above passage.
A. No, because the organisation works for a very noble cause.
B. No, because it is not mandatory to register a will.
C. Yes, because the will is not registered.
D. Both a and b.

77. A is the testator of an unregistered will. In his will, he has allotted 60% of his wealth to his son,
B and the rest 40% to his daughter, C. After A’s death, C contends that B forced A to give 60% to B
and only 40% to C. Can C challenge the execution of the will in this circumstance?
A. Yes, Since the will is unregistered.
B. No, because the daughters get lesser property in comparison to sons.
C. Yes, since the will seems to have an element of coercion.
D. Both a and c.

78. In the above case, the burden to prove that the will was made due to B’s force will lie upon
whom of the following?
A. C will have to prove the objection.
B. B will have to prove his innocence.
C. No one, since the will is not registered.
D. Cannot be determined.

79. A is the testator of a registered will. In his will, he has allotted 60% of his wealth to his son, B
and the rest 40% to his daughter, C. After A’s death, C contends that the will is not executable
because it contains the signature of only one witness. Can C challenge the execution of the will in
this circumstance?
A. Maybe
B. No
C. Yes
D. Cannot be determined

80. A is the testator of an unregistered will. In his will, he allotted some of his properties to an
unknown person. A’s son wanted to challenge the will. As per the above passage, decide whether
A’s son can challenge it or not?
A. No, because it is a registered will.
B. No, because the properties were allotted to an unknown person.
C. Yes, because registration of the will is optional.
D. None of the above.

Website: www.acingclat.com
YouTube: https://www.youtube.com/channel/UCCxsyMbrGVvbYu6Ua2eTb0A
Contact No. : +91-9166693196
DIRECTIONS (Q81 - Q90): Read the paragraph and answer the following:
Assault is an act of one person causing another person to reasonably apprehend the infliction of
criminal force against him. In Criminal Law, assault is considered as an offence and in Common
Law, it is considered as a tort. But, in both the cases assault is considered as an act of threat by one
person to cause harm to another person.
Assault is considered as a tort under the common law regime. Assault as a tort will be completed
when one person by his act creates an apprehension in the mind of the other person that the
former will be committing battery against the latter. Therefore, assault can be construed as a tort if
there exists a prima facie ability to do the harm. It is thus considered that assault precedes battery,
but it is not essential that every battery should include an assault. Assault becomes battery if the
person apprehending the harm is inflicted with that harm as apprehended by him. Therefore, till
the act remains to be a threat, it is considered as assault.
Should battery necessarily include assault?
No, if the victim was not in a position to apprehend as he was not aware of the use of criminal force
by the accused against him, then battery will not include assault.

81. A made an act of showing clenched fist towards B. As per the above passage, which of the
following statements is correct?
A. A will be held liable for assault only.
B. A will be held liable for battery only.
C. A will be liable for both assault and battery.
D. A will not be liable.

82. A made an act of throwing water at B. However, the water doesn’t reach B. As per the above
passage, which of the following statements is correct?
A. A will be held liable for assault only.
B. A will be held liable for battery only.
C. A will be liable for both assault and battery.
D. A will not be liable.

83. A made an act of throwing water at B. And B is completely drenched by the water thrown at
him. As per the above passage, which of the following statements is correct?
A. A will be held liable for assault only.
B. A will be held liable for battery only.
C. A will be liable for both assault and battery.
D. A will not be liable.

84. A was once passing by and B suddenly threw water at him. And A is completely drenched by
the water thrown at him. As per the above passage, which of the following statements is correct?
A. A will be held liable for assault only.
B. A will be held liable for battery only.
C. A will be liable for both assault and battery.
D. A will not be liable.

Website: www.acingclat.com
YouTube: https://www.youtube.com/channel/UCCxsyMbrGVvbYu6Ua2eTb0A
Contact No. : +91-9166693196
85. A made an act of putting an unloaded pistol on B’s head and threatened him. As per the above
passage, which of the following statements is correct?
A. A will be held liable for assault only.
B. A will be held liable for battery only.
C. A will be liable for both assault and battery.
D. A will not be liable.

86. A is about to sit on the chair and the chair is pulled by B. However, A doesn’t fall. As per the
above passage, which of the following statements is correct?
A. B will be held liable for assault only.
B. B will be held liable for battery only.
C. B will be liable for both assault and battery.
D. B will not be liable.

87. Which of the following statements can be correctly inferred from the above passage?
A. Assault succeeds Battery.
B. Battery succeeds Assault.
C. Every Battery includes Assault.
D. All of the above.

88. X loosens the muzzle of a ferocious dog knowing that the dog will cause harm to Y. However,
the dog doesn’t harm Y. As per the above passage, which of the following statements is correct?
A. X will be held liable for assault only.
B. X will be held liable for battery only.
C. X will be liable for both assault and battery.
D. X will not be liable.

89. A lays a trap for B and a few days later B falls into it. As per the above passage, which of the
following statements is correct?
A. A will be held liable for assault only.
B. A will be held liable for battery only.
C. A will be liable for both assault and battery.
D. A will not be liable.

90. One day, A sees B walking to his home. A suddenly comes from behind B and pushes B. As per
the above passage, which of the following statements is correct?
A. A will be held liable for assault only.
B. A will be held liable for battery only.
C. A will be liable for both assault and battery.
D. A will not be liable.

DIRECTIONS (Q91 - Q95): Read the paragraph and answerthe following:


Consensus ad idem means meeting of minds, mostly used in contract law, which refers to the fact
that there is an agreement among the parties to contract. It simply means that there exists a mutual

Website: www.acingclat.com
YouTube: https://www.youtube.com/channel/UCCxsyMbrGVvbYu6Ua2eTb0A
Contact No. : +91-9166693196
agreement among all parties to a contract. The agreement is one of the essential ingredients that
constitute a contract. If the parties do not reach a stage of mutual assent then a valid contract can
not be formed. The phrase is commonly used to refer to a situation of mutual understanding in the
formation of a contract over the same thing. The formation of a valid contract consists of certain
essential characteristic features, the agreement being one of them. Therefore, it can be understood
in a way that in the absence of meeting of minds, a contract so formed is void ab initio (i.e. null and
void from the very beginning). Meeting of minds is very essential to avoid any kind of
misunderstanding and confusion in the later stages of a contract and to ensure proper execution of
the purpose of entering into a contract. Thus, an agreement is a key feature that can make or break
a contract or a legal relation, it is therefore very necessary that the parties are on the same page as to
material facts, and contingencies involved in a contract.
It is clear from the above flow chart that agreement to all the terms of a contract are very essential
for a valid contract to be formulated. This phrase of Consensus ad idem originated from the fact
that contracts are rendered void if the parties are not on the same tangent when it comes to terms
and conditions that constitute a contract. Meeting of minds is therefore very necessary. It does not
completely eradicate the possibility of difference in opinion but simply means that all the different
opinions shall be clearly discussed and accommodated in a manner that does not overlook the
interests of all parties to a contract. It focuses on the removal of any sort of confusion, that may,
later on, lead to conflicts. Hence, mutual agreement is essential to keep away any conflicting
situation that may arise from a misunderstanding when the contract was formulated.

91. A who owns two horses named Rajhans and Hansraj. A selling horse Rajhans to B. B thinks
that he is purchasing horse Hansraj. A and B enter into an agreement. As per the above passage,
which of the following statements is correct?
A. The agreement is valid.
B. The agreement is void.
C. The agreement is voidable.
D. Cannot be determined.

92. A was willing to sell his house to B in Rs. 1,00,000. B wanted to purchase it for Rs. 80,000 and
conveyed this to A. A agreed and they entered into an agreement. As per the above passage, which
of the following statements is correct?
A. The agreement is valid.
B. The agreement is void.
C. The agreement is voidable.
D. Cannot be determined.

93. A agrees to purchase a painting from B, thinking that the painting is a masterpiece of C. B
agrees to sell the painting to A, knowing that the painting was actually created by D. As per the
above passage, which of the following statements is correct?
A. The agreement is valid.
B. The agreement is void.
C. The agreement is voidable.
D. Cannot be determined.

Website: www.acingclat.com
YouTube: https://www.youtube.com/channel/UCCxsyMbrGVvbYu6Ua2eTb0A
Contact No. : +91-9166693196
94. A was a French jeweller, who came to India to buy unique gems and stones. He found the
perfect stone that he was looking for and asked the Indian jeweller to order 10 pieces of the same
stone. Due to language issues, the Indian jeweller ordered 50 pieces and is asking for the payment of
all the stones as per their agreement. As per the above passage, which of the following statements is
correct?
A. The agreement is valid.
B. The agreement is void.
C. The agreement is voidable.
D. Cannot be determined.

95. A goes to a bakery to buy whole wheat bread, preferably of X brand, but the shopkeeper gave
him the bread manufactured by Y brand. A paid and took the bread. As per the above passage,
which of the following statements is correct?
A. The agreement is valid.
B. The agreement is void.
C. The agreement is voidable.
D. Cannot be determined.

DIRECTIONS (Q96 - Q100): Read the paragraph and answerthe following:


I agree with the call for uniformity in grounds for granting maintenance and alimony. But I say that
the petitions are misdirected so far as divorce is concerned. The petitions should instead ask for
scrapping all grounds for divorce, across communities. Divorce by mutual consent is a
self-explanatory phrase. It is also the simplest manner of ending a marriage. In case of a one-sided
divorce, however, all laws list certain grounds. A person needs to prove at least one legal ground for
the court to order such a divorce. Some grounds are common across all laws, while some differ. A
person can get a divorce if they prove to the court that their spouse was at fault in any of these ways.
True, the Supreme Court has recognised a special ground for divorce, not based on the fault theory.
It is the ground of ‘irretrievable breakdown of marriage’. But only the Supreme Court may grant a
divorce on this ground and that too when it is convinced ‘that there is absolutely no chance of the
marriage surviving’.

The problem here is the subjectivity involved in a decision on divorce. There can be several layers to
why a person no longer wishes to continue with their marriage. As such, how can a judge be
qualified enough to decide whether a person is unhappy enough in a marriage? I say our divorce
laws are violative of Indian constitutional principles too. They restrict a person’s fundamental right
to life and personal liberty and with no reasonable basis. Why should a person need to fight a battle
for divorce if they do not want to live with their spouse anymore? I can understand a legal contest
over custody of children and maintenance amounts. But contesting a divorce is like forcing one’s
spouse to stay married against their will.

Unfortunately, Indian courts and the legislature have been too conservative on divorce. For
instance, triple talaq in Muslim law was an exception to the current laws on divorce. It allowed
unilateral, no-cause divorce. But triple talaq has now been criminalised. I agree that the triple talaq

Website: www.acingclat.com
YouTube: https://www.youtube.com/channel/UCCxsyMbrGVvbYu6Ua2eTb0A
Contact No. : +91-9166693196
system was unconstitutional. It only allowed the privilege to men and discriminated against
women. Still, the right thing to do may have been for the legislature to extend the right to both
sexes, in all religions. The legislature still has that prerogative. I can only hope that the grounds
pleaded here for having no grounds for divorce at all, weigh with our lawmakers.

96. As per the above passage, which of the following is the main argument of the author?
A. That judiciary should remain the final authority to grant divorce.
B. That Triple Talaq was constitutional.
C. That the only ground for marriage should be ‘irretrievable breakdown of marriage’.
D. That all the grounds for divorce must be scrapped.

97. Which of the following statements are correct as per the author of the above passage?
A. The Triple Talaq system was better than the current divorce laws.
B. Divorce laws are violative of fundamental rights.
C. Divorce laws are constitutional.
D. Legal battles over child custody are unconstitutional.

98. Which of the following statements can be correctly assumed from the above passage?
A. Indian legislature is not in favour of divorce.
B. Indian judiciary is not in favour of divorce.
C. Both a and b.
D. None of the above.

99. A and B are husband and wife respectively. A was not happy with his marriage to B and wanted
a divorce. They had a son C. After their divorce (with mutual consent), A wanted custody of C.
Can A sue B for C’s custody? Answer as per the author of the above passage.
A. No, because divorce laws are unconstitutional.
B. Yes, because the court can decide the case.
C. No, because it will restrict their fundamental rights.
D. Both a and c.

100. The author of the above passage is most likely to agree with which of the following situations?
A. A and B are husband and wife respectively. A wanted to get divorce and B did not. A
should be allowed to get out of the marriage without approaching the court.
B. A and B are husband and wife respectively. A wanted to get divorce and B did not. They
should approach their Family court to fight the divorce.
C. A and B are husband and wife respectively. A wanted to get divorce and B did not. They
should settle their problems by themselves and consider avoiding divorce.
D. A and B are husband and wife respectively. A wanted to get divorce and B did not. They
should practice triple talaq and get divorced.

DIRECTIONS (Q101 - Q105): Read the paragraph and answerthe following:


When property, land or any other commodity is used as collateral to borrow money or to take a
loan from a lender, it is known as Mortgage. Discussed below are the different types of mortgages:

Website: www.acingclat.com
YouTube: https://www.youtube.com/channel/UCCxsyMbrGVvbYu6Ua2eTb0A
Contact No. : +91-9166693196
● Simple Mortgage: In such type of mortgage, the borrower needs to sign an agreement
stating that if he/she is unable to pay back the borrowed amount in specified time duration,
then the lender can sell the property to anyone to get his money back
● Mortgage by Conditional Sale: Under such mortgage, the lender can put a certain number
of conditions which the borrower must follow in terms of repayment. These conditions
may include the sale of the property if there is a delay in the monthly instalments, an
increase in the rate of interest due to delay in repayment, etc.
● English Mortgage: In this type of mortgage, the borrower has to transfer the property in the
name of the lender at the time of taking money, at a condition that the property would be
transferred back to the borrower once the complete amount is paid back
● Fixed-Rate Mortgage: When the lender assures the borrower that the rate of interest will
remain the same throughout the loan period is called Fixed-Rate Mortgage
● Usufructuary Mortgage: This kind of mortgage gives a benefit to the lender. The lender has
the right over the property for the due course of the loan period, he can put the property
on rent or use it for other purposes until the repayment of the amount. But the main rights
lie with the owner himself
● Anomalous Mortgage: A combination of different types of mortgages is called an
Anomalous Mortgage
● Reverse Mortgage: In this case, the lender lends money to the borrower on a monthly basis.
The entire loan amount is divided into instalments and the lender gives the borrower that
money in instalments
● Equitable Mortgage: In this type of mortgage, the title deeds of the property are given to
the lender. This is a common phenomenon in banking mortgage loans. It is done to secure
the property.

101. A man mortgaged his 2 BHK flat to a money lender in return for a sum of money and the
lender decided to let the flat on rent. This is an example of which type of mortgage?
A. Equitable Mortgage
B. Usufructuary Mortgage
C. English Mortgage
D. Fixed Rate Mortgage

102. If the lender sells the mortgaged property if the borrower is unable to repay the loan by the
decided date, this type of mortgage is called _________?
A. English Mortgage
B. Simple Mortgage
C. Sale Mortgage
D. Mortgage by Conditional Sale

103. A borrows Rs. 100000 from B. They agree that A will repay the whole amount by December
2021 at an interest rate of 3% and if he is unable to pay by then, the rate of interest will remain the
same. This is an example of which type of mortgage?
A. Equitable Mortgage
B. Usufructuary Mortgage

Website: www.acingclat.com
YouTube: https://www.youtube.com/channel/UCCxsyMbrGVvbYu6Ua2eTb0A
Contact No. : +91-9166693196
C. English Mortgage
D. Fixed Rate Mortgage

104. A borrows Rs. 100000 from B. They agree that B will give out money to A in 10 installments
of Rs. 10000 each. This is an example of which type of mortgage?
A. Equitable Mortgage
B. Simple Mortgage
C. Reverse Mortgage
D. Fixed Rate Mortgage

105. A borrows Rs. 100000 from XYZ bank. A mortgages his flat to the bank in exchange of the
money and the bank takes the title deeds of the flat to secure the property. This is an example of
which type of mortgage?
A. Equitable Mortgage
B. Simple Mortgage
C. Reverse Mortgage
D. Fixed Rate Mortgage

LOGICAL REASONING
DIRECTIONS (Q106 - Q110): Read the paragraph and answerthe following:
China, whose actions between April and May 2020 set off the crisis that threatened to destabilise
the entire region seems to have accepted that whatever military objectives it might have had at the
start, the price of achieving them would be heavier than it had perhaps estimated. The Indian
decision to capture the heights on the southern bank of Pangong Tso gave it a strategic advantage
and caught the People’s Liberation Army off guard. And the strong message from India that it
cannot be business as usual after China’s decision to violate all the bilateral agreements on
maintaining peace and tranquillity at the border, including the decision to ban Chinese companies
and scrutinise Chinese investment, no doubt played a role in disincentivizing Chinese adventurism
at the LAC. The signalling from the Chinese side at the end of the eighth round of talks for
disengagement provided the earliest hints that, in marked contrast to its usual strident posturing
with neighbours with whom it has boundary issues, Beijing was eager to step back from a military
crisis of its making. As both sides pull back their troops in a synchronised manner, the mood
should be one of sober and guarded optimism. It must be remembered that the Galwan clash, in
which India lost 20 soldiers, occurred immediately after an agreement to disengage. The timing of
the disengagement, after the thaw has set in, is itself surprising. Trust between the two sides is at
such a low ebb that India cannot afford to drop its guard. The lesson from Doklam is that
verification will be crucial to this process.

106. Which of the following most accurately expresses the main point of the passage?
A. The Chinese troops are not going to back off without a fight.
B. Indian troops have secured the border and the Chinese are in fear.
C. As China hints de-escalation, India must cautiously proceed and take lessons from the past.
Website: www.acingclat.com
YouTube: https://www.youtube.com/channel/UCCxsyMbrGVvbYu6Ua2eTb0A
Contact No. : +91-9166693196
D. India must not let off its guard against the Chinese till they do so.

107. What can be inferred from the passage?


A. The eight round of talks have ended.
B. The ninth round of talks have begun.
C. China has disengaged the border crisis.
D. India has disengaged the border crisis.

108. What can be inferred from the passage about China?


A. China has a reputation for friendly posture in dealing with border issues.
B. China has border skirmishes with several countries.
C. China seeks to back down quickly to de-escalate tensions at the border.
D. China staunchly stands up against the neighbour at the border issues.

109. What lesson do you think Doklam could have given the Indian troops?
A. To never back down even when China has disengaged completely
B. To not back down even when China hints disengagement in the talks
C. To attack China if it refuses to back down from the border.
D. To disengage and settle down immediately after China hints at disengagement at the
border.

110. What is the premise on which the author bases the strategy that India must not back down
even when China hints at disengagement?
A. The military had prophesied that China has betrayal plans.
B. A similar incident of Chinese backstabbing had occurred beforehand.
C. Chinese premier is a good friend of India.
D. China Exports a lot to India.

DIRECTIONS (Q111 - Q115): Read the paragraph and answerthe following:


What American export could have got the French all het up? No, it’s neither McDonalds, nor the
dubious pleasures of Hollywood, but, hold your artisanal baguette, social science theories from
American universities. From ministers to intellectuals and journalists, an influential section in
France has given voice to dark mutterings against theories of race, gender and postcolonialism —
which they see as importing an extreme woke scrutiny alien to French life. President Emmanuel
Macron has blamed such ideas for an “ethnicisation of the social question’’ that runs the risk of
dividing the country further. This culture war has resonance anywhere in the world, where the old
consensus cracks under the weight of the present. France might want American theories to stay
away from its “internal matter”, but that’s the thing with ideas: It’s hard to stop them from
travelling.

111. What can be inferred from the passage?


A. American social science theories are accepted and appreciated in France.
B. French president is opposed to the exchange of culture with different countries.
C. A theory has the capacity to cause divisions in the country.

Website: www.acingclat.com
YouTube: https://www.youtube.com/channel/UCCxsyMbrGVvbYu6Ua2eTb0A
Contact No. : +91-9166693196
D. French universities teach the social science theories of American universities.

112. What is the main point as expressed in the passage?


A. French are against the Americans.
B. French are against the exchange of American universities with the French.
C. The French are against the import of social science theories of American universities.
D. One cannot stop the exchange of cultures and ideas

113. How do French react when they are het up as inferred from the passage?
A. They become distraught and agitated
B. They get all sad and gloomy
C. They tend to delve deep in thoughts
D. They are relieved and happy

114. What is the premise which motivates France to want to keep American theories to stay away?
A. The social science theories of America are depressing
B. The social science theories of America are divisive.
C. The social science theories of America are more advanced than those of French
D. The social science theories of America are drafted by their government.

115. What is the assumption on which the French President makes his statement?
A. The country is already divided.
B. The country cannot be studying social sciences.
C. Social Sciences do not have the power to divide a country.
D. People in France are protesting against the government.

DIRECTIONS (Q116 - Q120): Read the paragraph and answerthe following:


When a government shows signs of paranoia, it has dangerous consequences. We are beginning to
see this happen. Political activists are tossed into jail these days under stringent preventive detention
laws and recently we saw journalists and an MP charged with ‘sedition’. What is going on? Does the
Prime Minister truly believe that the only way he can govern India is by crushing all dissent under
the jackboot? Does he not see that if Twitter is to be reprimanded for allowing offensive tweets, the
first accounts that it should close are those that belong to people ‘proud to be followed’ by him. He
only needs to check some of the tweets that the BJP’s troll army posts. Why should the most
powerful prime minister in Indian parliamentary history need to behave as if he is under an
existential threat from shadowy foes? Why should dissent be treated as sedition? Why should
protesters need to be categorised and labelled? It would have been better if the Prime Minister had
not himself made clear that he approves of this kind of repression.

116. What can be inferred about the prime minister from the passage?
A. That he s a selfless man devoted to the country
B. That he only cares for the Hindus of the country.
C. That he is very popular amongst the people of the country.
D. That he is against the criticism of his government.

Website: www.acingclat.com
YouTube: https://www.youtube.com/channel/UCCxsyMbrGVvbYu6Ua2eTb0A
Contact No. : +91-9166693196
117. What is the assumption of the author behind the BJP’s troll army posts?
A. That they had posted something that the government disapproved.
B. That they had posted something which spread communal hatred.
C. That they had posted something about the success of Ayodhya verdict.
D. That they had posted something in favour of the government.

118. What can be concluded from the passage?


A. That the government is very popular amongst the people.
B. That the prime minister approves the oppression of the people.
C. That the journalists who were arrested were personally biased.
D. That the government should stop curbing dissent.

119. Why does the author mention the case of twitter?


A. To show the inability of the government to control private players like twitter.
B. To show how powerful social media handles are.
C. To show how the government is trying to curb dissent on digital platform
D. To show bias of the government with respect to its own party members.

120. What is the main point of the passage?


A. That the government is anti-national
B. That the government favours its followers.
C. That the government is paranoid.
D. That the acts of curbing dissent is a paranoid activity.

DIRECTIONS (Q121 - Q125): Read the paragraph and answerthe following:


If history doesn’t follow any stable rules, and if we cannot predict its future course, why study it? It
often seems that the chief aim of science is to predict the future – meteorologists are expected to
forecast whether tomorrow will bring rain or sunshine; economists should know whether
devaluing the currency will avert or precipitate an economic crisis; good doctors foresee whether
chemotherapy or radiation therapy will be more successful in curing lung cancer. Similarly,
historians are asked to examine the actions of our ancestors so that we can repeat their wise
decisions and avoid their mistakes. But it never works like that because the present is just too
different from the past. It is a waste of time to study Hannibal’s tactics in the Second Punic War so
as to copy them in the Third World War. What worked well in cavalry battles will not necessarily be
of much benefit in cyber warfare. Science is not just about predicting the future, though. Scholars
in all fields often seek to broaden our horizons, thereby opening before us new and unknown
futures. This is especially true of history. Though historians occasionally try their hand at prophecy
(without notable success), the study of history aims above all to make us aware of possibilities we
don’t normally consider. Historians study the past not in order to repeat it, but in order to be
liberated from it. Each and every one of us has been born into a given historical reality, ruled by
particular norms and values, and managed by a unique economic and political system. We take this
reality for granted, thinking it is natural, inevitable and immutable. We forget that our world was
created by an accidental chain of events, and that history shaped not only our technology, politics

Website: www.acingclat.com
YouTube: https://www.youtube.com/channel/UCCxsyMbrGVvbYu6Ua2eTb0A
Contact No. : +91-9166693196
and society, but also our thoughts, fears and dreams. The cold hand of the past emerges from the
grave of our ancestors, grips us by the neck and directs our gaze towards a single future. We have felt
that grip from the moment we were born, so we assume that it is a natural and inescapable part of
who we are. Therefore, we seldom try to shake ourselves free, and envision alternative futures.
Studying history aims to loosen the grip of the past. It enables us to turn our head this way and
that, and begin to notice possibilities that our ancestors could not imagine, or didn’t want us to
imagine. By observing the accidental chain of events that led us here, we realise how our very
thoughts and dreams took shape – and we can begin to think and dream differently. Studying
history will not tell us what to choose, but at least it gives us more options.

121. Based on the passage, which of the following would be the most appropriate course of action
for citizens to learn history?
A. British names of streets in India should not be changed
B. Every street in India should display a plaque that lists all its previous names
C. Names of Indian streets should be based on suggestions generated through an opinion poll
D. Names of Indian streets should be periodically changed
122. Which of the following can be inferred from the passage?
A. We must not follow exactly what our ancestors did
B. History tells us what we must do in the face of options
C. Cavalry and cyber warfare are totally different
D. History is an exact science
123. Which of the following options is the most closely related to the passage?
A. History, unlike Physics, does not help predict future
B. History deals with long time periods
C. History documents the past events related to specific people
D. History has the potential to make us eclectic
124. Which of the following statements, if true, contradicts the view of the author?
A. A historian successfully predicted a political crisis based on similar events of the last century
B. A tribe in Andaman learns to predict epidemic outbreaks by listening to the stories of how
their ancestors predicted the past outbreaks
C. Both of these
D. None of these
125. Which of the following is true in the context of the passage?
A. History shapes nothing more than our thoughts, fears and dreams
B. We exist independent of our history
C. History does not follow rigid rules
D. Our ancestors had precise and perfect knowledge

DIRECTIONS (Q126 - Q130): Read the information given below to answer the
questions.
Mediators recognize that the law is generally in place to uphold mediator- client confidentiality,
however, there are situations that may occur where the mediator is under obligation to break the
confidence. This obligation can vary depending upon where the mediator is practising and it may
occur as a result of their employment contract or of the law. Where such an issue is with their

Website: www.acingclat.com
YouTube: https://www.youtube.com/channel/UCCxsyMbrGVvbYu6Ua2eTb0A
Contact No. : +91-9166693196
client; however, in situations where the factors under consideration are particularly urgent, it is
accepted that this cannot always be the case. Legitimate breaches of confidentiality relate to
circumstances where the information the client has shared relates to anti-national activities;
information of this nature must be reported. There are other circumstances where breaching
confidentiality may be considered legitimate, for example, in the case of serious crime or suspected
child abuse. Mediation service providers and mediators practising independently have their own
boundaries but must agree this contractually with their client at the onset of the client-council
relationship

126. Being obliged to break confidentiality in a mediator-client relationship is always a direct result
of the law.
A. Definitely True: The statement follows logically from the information contained in the
passage.
B. Definitely False: The statement is logically false from the information contained in the
passage.
C. Probably False: The statement is more likely to be false than true, but not definitely false
beyond a reasonable doubt, based solely on the information in the passage.
D. Data Inadequate: It is not possible to determine whether the statement is true or false
without further information.

127. When a client reports their involvement in a serious crime, their Mediators are legally obliged
to report this?
A. Definitely True: The statement follows logically from the information contained in the
passage.
B. Definitely False: The statement is logically false from the information contained in the
passage.
C. Probably False: The statement is more likely to be false than true, but not definitely false
beyond a reasonable doubt, based solely on the information in the passage.
D. Data Inadequate: It is not possible to determine whether the statement is true or false
without further information.

128. Mediators have some flexibility regarding what they deem serious enough to lead them to
breach confidentiality.
A. Definitely True: The statement follows logically from the information contained in the
passage.
B. Definitely False: The statement is logically false from the information contained in the
passage.
C. Probably False: The statement is more likely to be false than true, but not definitely false
beyond a reasonable doubt, based solely on the information in the passage.
D. Data Inadequate: It is not po

129. Most Mediators agree with the conditions that the law places on them relating to breaching
client confidentiality.

Website: www.acingclat.com
YouTube: https://www.youtube.com/channel/UCCxsyMbrGVvbYu6Ua2eTb0A
Contact No. : +91-9166693196
A. Definitely True: The statement follows logically from the information contained in the
passage.
B. Definitely False: The statement is logically false from the information contained in the
passage.
C. Probably False: The statement is more likely to be false than true, but not definitely false
beyond a reasonable doubt, based solely on the information in the passage.
D. Data Inadequate: It is not possible to determine whether the statement is true or false
without further information.

130. If a mediator breaches his client's confidence about a serious issue without first informing
their client of their intentions, they are breaking the law.
A. Definitely True: The statement follows logically from the information contained in the
passage.
B. Definitely False: The statement is logically false from the information contained in the
passage.
C. Probably False: The statement is more likely to be false than true, but not definitely false
beyond a reasonable doubt, based solely on the information in the passage.
D. Data Inadequate: It is not possible to determine whether the statement is true or false
without further information.

DIRECTIONS (Q131 - Q135): Read the following passage to answer the questions.
A body to prepare the Constitution was finally set up under the 1946 Cabinet Mission
Programme. It was a proposal to keep British India undivided, and planned to do this by grouping
provinces by religion, and restricting the Federal Government's involvement in national issues such
as external policy. However, it left critical issues such as the status of the princely states unresolved.
The Muslim League boycotted the Constituent Assembly's early sessions and called for assemblies
to produce texts for two different countries. With time, under the 1947 Indian Independence Act,
the Assembly acquired legal status and formed two distinct dominions – India and Pakistan. These
deliberations were held between December 1946 and November 1949. This was during the civil
war, one of the largest human migrations, and amidst the incorporation of almost 600 princely
states into the India Union. This body was elected based on elections to the provincial legislatures
from 1945 to 1946. Only an approximate quarter to a fifth of the adult population were registered
to vote during these elections. There were also close to a hundred delegates representing the
princely states, who were selected by a consultation process. Moreover, the creation of Pakistan and
exit of the Muslim League left the Indian National Congress with an overwhelming majority.
Despite this, however, the body emerged as a remarkable forum for the articulation of intellectual
disagreements and contrasting viewpoints.

131. Which of the following can be inferred from the passage?


A. The Indian National Congress used its majority to suppress any opposing viewpoints
B. Despite having majority in the Constituent Assembly, the Indian National Congress did
not prevent opposing viewpoints
C. The exit of the Muslim League ensured that the only voice was that of the Indian National
Congress

Website: www.acingclat.com
YouTube: https://www.youtube.com/channel/UCCxsyMbrGVvbYu6Ua2eTb0A
Contact No. : +91-9166693196
D. The delegates of the princely states in the Constituent Assembly disrupted the Assembly’s
proceedings
132. Which of the following provides the best explanation for why the Muslim League boycotted
the early sessions of the Constituent Assembly?
A. The Muslim League felt that the Indian National Congress was too biased in favour of the
Muslims
B. The Muslim League felt that they would be able to draft a better text for India than the
Indian National Congress
C. The Muslim League felt that it would be better if two different constituent assemblies were
constituted to draft constitutions for two different countries.
D. The Muslim League felt that the Constituent Assembly only represented the interests of
businessmen
133. What is the main point of the passage?
A. That the Constituent Assembly had different viewpoints even though it was not fully
representative of the population of British India
B. That the Constituent Assembly was not representative of British India, and hence did not
have any legitimate authority to frame the Constitution
C. That the Constituent Assembly included too many representatives from the Indian
National Congress
D. That the Constituent Assembly included too many representatives from the princely states
134. Despite the limitations mentioned in the passage, which of the following could be a reason for
the multiplicity of viewpoints in the Constituent Assembly?
A. The rules of the Constituent Assembly provided that all groups, no matter how small, had
adequate voting power in the Assembly’s decision-making
B. There were a number of procedural safeguards which ensured that all groups, no matter
how small, had a chance to air their viewpoints
C. Both of these
D. None of these
135. Which of the following, if true, would weaken the author's main argument the most?
A. Presenting differing viewpoints is not enough to provide legitimate authority to frame the
Constitution to such a non-representative Constituent Assembly.
B. It is not necessary for the Constituent Assembly to have been representative of the adult
population of British India for it to have differing viewpoints.
C. The Constituent Assembly was completely within the control of the Indian National
Congress but it did provide differing viewpoints.
D. A body that was not fully representative of the adult population of British India can never
provide opportunities for debate and differing viewpoints.

Website: www.acingclat.com
YouTube: https://www.youtube.com/channel/UCCxsyMbrGVvbYu6Ua2eTb0A
Contact No. : +91-9166693196
QUANTITATIVE TECHNIQUES

Directions 136 to 140: A room having the dimensions as mentioned in the chart below
has a table, a bed and a chair. There are three books kept on the table (side by side) and
also a lamp having a radius, 3.5 inch. Now, on the basis of information given below answer
the following.

136. What would be the area of the room which is not covered by any object as mentioned above?
A. 176.5 sq. ft.
B. 178 sq. ft.
C. 367 sq. ft.
D. 245.5 sq. ft.

137. What is the area of the table which is uncovered by any of the objects?
A. 1656 sq. inch
B. 1617.52 sq. inch
C. 1473.52 sq. inch
D. 1512 sq. inch

138. How many tables are required to cover the floor surface of the room completely?
A. 18
B. 11
C. 21
D. Data inadequate

139. If the entire room is to be filled by books having the same dimensions as lying on the table
then how many books are required?
A. 2,00,736
B. 20,736
C. 11,736
D. 17,522

Website: www.acingclat.com
YouTube: https://www.youtube.com/channel/UCCxsyMbrGVvbYu6Ua2eTb0A
Contact No. : +91-9166693196
140. How many chairs can be put inside the room so as to cover the entire floor area of the room?
A. 40
B. 21
C. 43
D. 18
Directions (141 to 145): Answer the following question based on the information given
below:
Every year, a survey of 1000 people is conducted by the World Health Organization (WHO).
WHO found that in the year 2005, 2006, 2007, 2008 and 2009 the percentage of people affected by
malaria were 30%, 40%, 30%, 20% and 45% respectively. WHO also found that every year out of the
affected people 60% were students, 10% were house-wives and 30% were drivers. The number of
house-wives, students and drivers were in the ratio 20 : 11 : 9, every year.

141. In the year 2007, find the number of house-wives affected by malaria?
A. 60 B. 30
C. 50 D. 110

142. In the year 2009, find the number of drivers who were not affected by malaria?
A. 110 B. 125
C. 415 D. 90

143. What is the difference in the number of students affected and not affected by malaria in the
year 2006?
A. 205 B. 35
C. 200 D. 240

144. Find the ratio of the number of house-wives affected by malaria in the year 2005 to that
affected by malaria in the year 2008.
A. 5:3 B. 9:4
C. 3:2 D. 2:1

145. Which year had the maximum number of students not affected by malaria?
A. 2005 B. 2006
C. 2007 D. 2008

Directions (146 to 150): Study the following information caref ully and answer the
questions given below it.
There are 5200 employees in an organisation working in various departments viz. HR, Marketing,
Finance, IT and Legal. The employees in the various departments are either Graduates or
Postgraduates. 25% of the total number of employees are from HR departments 12% of the total
number of employees are from Marketing department. 45% of the total number of employees in
the HR department are Graduates. 50% of the total number of employees in the Marketing
department are Postgraduates. 18% of the total number of employees in the organization are from
Website: www.acingclat.com
YouTube: https://www.youtube.com/channel/UCCxsyMbrGVvbYu6Ua2eTb0A
Contact No. : +91-9166693196
Finance department out of which 75% are Postgraduates. 546 employees from IT department are
Postgraduates. 15% of the total number of employees in the organisation are in Legal department.
60% of the total number of employees in the Legal department are Graduates.

146. What is the total number of employees in HR, Finance and Legal departments together?
A. 3,484
B. 2,860
C. 3,640
D. 3,016
147. What is the respective ratio of the number of Postgraduates in the Legal department to the
number of Postgraduates in the HR department?
A. 8:11
B. 3:5
C. 24:55
D. 12:55
148. The number of Graduates in the Finance department is what percent of the total number of
employees in the organisation?
A. 4.5
B. 5.5
C. 3
D. 5
149. What is the total number of Post Graduates in the organisation from all the departments
together?
A. 2,597
B. 2,500
C. 2,867
D. None of These
150. What is the total number of employees in the IT department?
A. 1,014
B. 1,300
C. 1,560
D. 1,650

END OF THE SAMPLE CLAT MOCK TEST - AC001

Website: www.acingclat.com
YouTube: https://www.youtube.com/channel/UCCxsyMbrGVvbYu6Ua2eTb0A
Contact No. : +91-9166693196
Acing CLAT’s One Year & Two Year Courses

Website: www.acingclat.com
YouTube: https://www.youtube.com/channel/UCCxsyMbrGVvbYu6Ua2eTb0A
Contact No. : +91-9166693196
Acing CLAT’s Test Series & Mentorship Session

Website: www.acingclat.com
YouTube: https://www.youtube.com/channel/UCCxsyMbrGVvbYu6Ua2eTb0A
Contact No. : +91-9166693196
ANALYSIS CLAT TEST CODE - AC001

ENGLISH LANGUAGE

1. A; In the last part of 1st para "Because labor rates are highly visible, managers can easily compare
their company’s rates with those of competitors"

2. D; Process of elimination Option D is in line with that of managers thinking

3. B; In 1st para "But labor costs and labor rates are not in fact the same: one company could pay its
workers considerably more than another and yet have lower labor costs if that company’s
productivity were higher due to the talent of its workforce, the efficiency of its work processes, or
other factors."It shows that High labor rates might be the problem for all the companies. Might be
for few but not all.

4. A; Question talks about "What author Suggests ABOUT the suggestions of consultancy
In last lines of the passage "Finally, to the extent that changes in compensation create new
problems" It will create new problems--Thus failing in bringing the intended change(Goal)

5. D; In last lines of the passage "so managers are more likely to find such advice from consultants
palatable"

6. D; It is an easy question.

7. to 14.

Para1: the main theme---the role of grazers(zooplankan) in controlling the amount of plantonic
algae--- which form the three theories we will discuss about first theory: grazer control were merely
based on observations of negative correlations between algal and zooplankton numbers. the
absence of grazers in areas of high phytoplankton concentration -lead to --->Hardy’s principle of
animal exclusion

Para2: flaw in para1’s study: consider only algae the size/deemphasize the role of grazers/grazing is
just a minor component of models to predict alga population
-->….second theory---researchers stress the importance of environmental factors(temperature、light
、water)

Para3: use the idea of “grazing pressure” as third theory studies of Hargrave & Geen--- >estimate
natural community grazing rate derive the estimates of grazing pressure
-->Haney, record zooplankton grazing rates in natural field conditions by using different species&
season to compare
-->lead to the conclusion---“grazers can exert significant pressure on phytoplankton population”
correspond to the pre-thinking summary: the author propose three kinds of theories, which he
introduces one by one separately in para1-3, to explain the factors that will affect the role of
grazers(zooplankton) in controlling the amount of planktonic algae.

7. A; The converse observation, of the absence of grazers in areas of high phytoplankton


concentration, led Hardy to propose his principle of animal exclusion, which hypothesized that
phytoplankton produced a repellent(causing disgusting or able to repel) that excluded grazers from
regions of high phytoplankton concentration. This was the first suggestion of algal defenses against
grazing. (A) give an example of one theory about the interaction of grazers and phytoplankton
….correct. Hardy's principle of animal exclusion ….which hypothesized that phytoplankton
produced a repellent that excluded grazers from regions of high phytoplankton concentration.

8. D; (D) make verifiable correlations of cause and effect between zooplankton and phytoplankton
numbers ….correct. see sentences in para1&2: The first theories of such grazer control were merely
based on observations of negative correlations between algal and zooplankton numbers. Perhaps
the fact that many of these first studies considered only algae of a size that could be collected in a
net (net phytoplankton), a practice that overlooked the smaller phytoplankton (nannoplankton)
that we now know grazers are most likely to feed on, led to a de-emphasis of the role of grazers in
subsequent research.…..beside the negative correlation in first theory mention above, we should
also consider “the role of grazers” ,which could also be a factor that affect algal and zooplankton
numbers, to make the correlation more verifiable.

9. B; Zooplankton exclusion is unrelated to phytoplankton population density. ….correct


If you doesn’t think thoroughly about the logic, it's quite easy for you to get trapped, and the logic
present as below: Hardy’s principle of animal exclusion/zooplankton exclusion - hypothesize that
“phytoplankton produced a repellent that excluded grazers from regions of high phytoplankton
concentration”. regions of high phytoplankton concentration -this concern about the population
density of phytoplankton. Thus we know: if…..……“zooplankton exclusion”—grazer repellent-- is
related to ”population density of phytoplankton”….. is true, -- then it could strengthen Hardy’s
principle of animal exclusion and further… if…..……“zooplankton exclusion”—grazer repellent-- is
UNrelated to ”population density of phytoplankton”….. is true, -- then it would weaken/put into
question Hardy’s principle of animal exclusion

10. C; The potential magnitude of grazing pressure on freshwater phytoplankton has only recently
been determined empirically. Studies by Hargrave and Geen estimated natural community grazing
rates by measuring feeding rates of individual zooplankton species in the laboratory and then
computing community grazing rates for field conditions using the known population density of
grazers. The high estimates of grazing pressure postulated by these researchers were not fully
accepted, however, until the grazing rates of zooplankton were determined directly in the field, by
means of new experimental techniques. Using a specially prepared feeding chamber, Haney was
able to record zooplankton grazing rates in natural field conditions. In the periods of peak
zooplankton abundance, that is, in the late spring and in the summer, Haney recorded maximum
daily community grazing rates, for nutrient-poor lakes and bog lakes, respectively, of 6.6 percent
and 114 percent of daily phytoplankton production. Cladocerans had higher grazing rates than

Website: www.acingclat.com
YouTube: https://www.youtube.com/channel/UCCxsyMbrGVvbYu6Ua2eTb0A
copepods, usually accounting for 80 percent of the community grazing rate. These rates varied
seasonally, reaching the lowest point in the winter and early spring. Haney’s thorough research
provides convincing evidence that grazers can exert significant pressure on the phytoplankton
population.

11. D; Haney...correct

12. C; collect phytoplankton of all sizes before analyzing the extent of phytoplankton
concentration ....correct

13. D; see sentences in para3:


Studies by Hargrave and Geen estimated natural community grazing rates by measuring feeding
rates of individual zooplankton species in the laboratory and then computing community grazing
rates for field conditions using the known population density of grazers. They estimated the
natural community grazing rates of zooplankton by using laboratory data concerning the grazing
rates of individual zooplankton species. …correct. Studies by Hargrave and Geen estimated natural
community grazing rates by measuring feeding rates of individual zooplankton species in the
laboratory

14. D; Using a specially prepared feeding chamber, Haney was able to record zooplankton grazing
rates in natural field conditions. In the periods of peak zooplankton abundance(zooplankton the
highest), that is, in the late spring and in the summer, Haney recorded maximum daily community
grazing rates, for nutrient-poor lakes and bog lakes, respectively, of 6.6 percent and 114 percent of
daily phytoplankton production. Cladocerans had higher grazing rates than copepods, usually
accounting for 80 percent of the community grazing rate. These rates varied seasonally, reaching
the lowest (grazing rates )point in the winter and early spring. Haney’s thorough research provides
convincing evidence that grazers can exert significant pressure on phytoplankton population.
….correct, best match to the calculation below: zooplankton numbers---highest----late spring 5 &
summer 6-7-8; March’s number<June’s number; grazing rate---lowest---winter 12-1-2 & early
spring 3; March’s grazing rate< June’s grazing rate

15. B R argued on something, Subsequent research made R to believe something else, based on
that belief R modifies her hypothesis. Hence, B is correct. As it clearly depicts that R took a
position and subsequent research forced her to change her hypothesis.

16. C; The passage indicates that Robertsonas research in Kenya caused her to change her mind
regarding which of the following? Ans- caused her to changer her mind regarding over
generalization about Africa. As seen through the passage, women in kenyan society were treated as
legal minors and sometimes as male property. This shows that gender played a prominent role in
determining authority in kenyan society even before colonialism. Since the question mentioned
"research in Kenya," we should zero in on the portion of the passage that discusses this research,
lines 18-24: "Subsequent research in Kenya convinced Robertson that she had overgeneralized
about Africa. Before colonialism, gender was more salient in central Kenya than it was in Ghana."
Compare this with R's initial theory (note that this is conveniently located near the beginning of
the passage, in lines 3-7): "Before colonialism, age was a more important indicator of status and

Website: www.acingclat.com
YouTube: https://www.youtube.com/channel/UCCxsyMbrGVvbYu6Ua2eTb0A
authority than gender in Ghana and in Africa generally." R has learned that even before
colonialism, gender was more important in Kenya than she'd thought it was in Africa generally.
This leads us straight to C.

17. B; The passage suggests that after conducting the research mentioned in the highlighted text,
but not before, Robertson would have agreed with which of the following about women’s
status and authority in Ghana? Ans- After conducting the research she would have agreed that she
over generalized and gender had played different role in ghana and kenya. Women in Ghana were
more independent as compared to women in other African countries (see Kenya), so colonialism
affected their lives the most, since they went from independent to legal minors. " "In contrast with
Ghana, where women had traded for hundreds of years and achieved legal majority (not unrelated
phenomena), the evidence regarding central Kenya indicated that women were legal minors".
Hence B.

18. D; The author of the passage mentions the status of age as a principle of social organization in
precolonial central Kenya in the highlighted text most likely in order to
Ans- “Thus” she revised her hypothesis after identifying all the stated factors. (Last line). Hence D.

19. D; summarize a number of scientific theories that attempt to explain why animal populations
do not exceed certain limits ="Although numbers of animals in a given region may fluctuate from
year to year, the fluctuations are often temporary and, over long periods, trivial. Scientists have
advanced three theories of population control to account for this relative constancy."

20. D; (D) make sufficiently accurate predictions about the effects of crowding - "One challenge for
density-dependent theorists is to develop models that would allow the precise prediction of the
effects of crowding."

21. A; As the number of foxes in Minnesota decrease, the growth rate of this population of foxes
begins to increase. This matches information (1).

22. D; According to the Wynne-Edwards theory as it is described in the passage, epideictic behavior
displays serve the function of: (D) regulating sexual activity -A third theory, proposed by
Wynne-Edwards and termed “epideictic,” argues that organisms have evolved a “code” in the form
of social or epideictic behavior displays, such as winter-roosting aggregations or group vocalizing;
such codes provide organisms with information on population size in a region so that they can, if
necessary, exercise reproductive restraint. "

23. B; The challenge posed to the Wynne-Edwards-theory by several studies is regarded by the
author with (B) qualified acceptance -"However, Wynne-Edwards’ theory, linking animal social
behavior and population control, has been challenged, with some justification, by several studies."

24. C; (C) The results of one study, for instance, have suggested that group vocalizing is more often
used to defend territory than to provide information about population density.-"However,
Wynne-Edwards’ theory, linking animal social behavior and population control, has been

Website: www.acingclat.com
YouTube: https://www.youtube.com/channel/UCCxsyMbrGVvbYu6Ua2eTb0A
challenged, with some justification, by several studies." The last sentence of the last paragraph states
that WE theory has been challenged by several studies . C goes ahead by giving an example.

25. B; B. is correct. The author in the beginning of passage refers to the ability of over water
migrants to travel on course as a "mysterious phenomenon". He goes on to talk about the inbuilt
biological clocks in the flying species, but in the very next line argues that this alone does not
explain how the birds navigate the ocean. Celestial navigation in birds also seems like a far-fetched
idea to the author. He concludes by saying that Some species have a magnetic sense.

26. A; A. is the correct answer. If the birds are traveling East (going right from North) and they are
blown far South by the storm, and they are traveling on compass sense alone then they would keep
going in the same direction i.e., East.

27. B; B. is the correct answer. The author says that some scientists thought that migrant animals
use celestial navigation to navigate over oceans. However, he thinks this to be a flawed explanation
as this would require the animals to have a "fantastic map sense".

28. D; Choice (D) matches the prediction and is correct. The first two sentences of the second
paragraph say that the gravitational forces are "greater" and that the oceans are "governed primarily
by Earth's gravity."

29. B; Choice (B) is a distortion and, therefore, is correct. The passage credits improvements in
cartography to advancing technology, not to stable sea levels.

30. C; Purpose - To explain/discuss The concept 'sea Level' and grow on that idea Closer options
A -> the passage is not focused on discussing the causes but the whole concept plus sea life is wrong
C-> correct - topographic variable = sea level

Website: www.acingclat.com
YouTube: https://www.youtube.com/channel/UCCxsyMbrGVvbYu6Ua2eTb0A
CURRENT AFFAIRS including GENERAL KNOWLEDGE

31. C; A total of 17 measures amounting to Rs. 6,28,993 crore were announced in the Economic
Relief Package.
32. A; 0.6%
33. A;

34. A; Ms. Sitharaman announced an expansion of the existing Emergency Credit Line Guarantee
Scheme (ECLGS) by ₹1.5 lakh crore. She also announced a new ₹7,500 crore scheme to guarantee
loans upto ₹1.25 lakh to small borrowers through micro-finance institutions.
35. A; The health sector component is aimed at scaling medical infrastructure targeting
underserved areas (i.e. non-metropolitan areas).
36. C; Both statements are correct.
37. A; Free one month tourist visa to 5 lakh tourists.
38. B; Tenure of Production-Linked Incentive (PLI) scheme for Large Scale Electronics
Manufacturing has been decided to be extended by one year i.e. till 2025-26.
39. B; corpus of 10 lakh
40. A; Children who have lost both parents or their lone surviving parent or their legal guardian or
adoptive parent due to Covid-19.

Website: www.acingclat.com
YouTube: https://www.youtube.com/channel/UCCxsyMbrGVvbYu6Ua2eTb0A
41. A; This corpus will be used to give a monthly stipend from 18 years of their age, for the next
five years and on reaching the age of 23 years, he or she will get the corpus amount as one
lump-sum for personal and professional use.
42. D; Kendriya Vidyalaya or in a private school as a day scholar
43. A; Statement I is incorrect as asked in the question. All children will be enrolled as a beneficiary
under Ayushman Bharat Scheme with a health insurance cover of Rs. 5 lakh.
44. A; The Fund is a public charitable trust with the Prime Minister as its Chairman. Other
Members include Defence Minister, Home Minister and Finance Minister.
45. A; Bal Swaraj (COVID-Care)
46. C; NCPCR is a statutory body set up in March 2007 under the Commissions for Protection of
Child Rights (CPCR) Act, 2005. It is under the administrative control of the Ministry of Women
& Child Development.
47. C; United Nations Human Rights Council
48. A;

49. A; Among the countries that abstained on the vote, along with India, were France, Italy, Japan,
Nepal, the Netherlands, Poland, and South Korea.
50. A; India is also the first non-Arab country to recognise Palestine Liberation Organisation
(PLO) as the sole representative of the Palestinian. India is also one of the first countries to
recognise the statehood of Palestine in 1988.
51. D; Israel is bordered by the Mediterranean Sea to its west, Egypt to the south, Jordan and Syria
to its east and Lebanon to the north.
Website: www.acingclat.com
YouTube: https://www.youtube.com/channel/UCCxsyMbrGVvbYu6Ua2eTb0A
52. A; About the declaration: This was problematic because in 1916, the British had secretly made
a deal with the French according to which after the war, the Arab territories would be divided and
Palestine would be in control of the British. Moreover, the British had also promised the ruler of
Mecca, Sharief Hussain, in 1915 that Hussain would rule over the region including Palestine if he
led an Arab revolt against the Ottomans, which he did in fact.
53. B; Hamas, a Sunni Islamist militant group won the elections in Palestine in 2006. In 2007,
Hamas defeated Fatah (political group that controlled the PLO) in 2007 in fighting that started in
2006. Hamas (which many consider a terrorist group) has been fighting with Israel with
particularly significant battles in 2008, 2012 and 2014.
54. C; All are correct
55. B; Israel claims the whole undivided city as its rightful capital. C is incorrect as Jerusalem is
important for Christianity as well.
56. A; The ONORC is not exactly a 2020 product. It was launched in August, 2019 while work on
the portability of ration cards had begun as early as April 2018.
57. C; Both are correct
58. B; The national platform runs on two supporting portals - Integrated Management of Public
Distribution System (IM-PDS) (impds.nic.in) and Annavitran (annavitran.nic.in).
59. A; Thirty-two states and Union Territories have already adopted the scheme. That is, except
Assam, Chhattisgarh, Delhi and West Bengal, all states have adopted the ONORC.
60. B; The beneficiary AAY household is entitled to 35 Kg of food grains per month. Each PHH
person is entitled to 5 Kg of foodgrains per month under the National Food Security Act.
61. B; The Act is in line with Goal Two of the Sustainable Development Goals set by the United
Nations General Assembly. Goal 2 seeks sustainable solutions to end hunger in all its forms by 2030
and to achieve food security.
62. D; All are correct
63. B; The fundamental right to life under Article 21 of the Constitution may be interpreted to
include the right to live with human dignity, having the right to food and other basic necessities.
64. A; Justice Raveendran Committee
65. C; All are correct
66. A; Malayalam is spoken in all the islands except Minicoy where people speak Mahl which is
written in Divehi script and is spoken in Maldives also
67. C; Both are correct
68. A; It is directly under the control of the Centre through an administrator. Amindivi Islands are
the northernmost while Minicoy island is the southernmost

LEGAL REASONING
69. (b) 70. (c) 71. (d) 72. (d) 73. (d) 74. (c) 75. (b) 76. (b) 77. (c) 78. (a) 79. (c) 80. (c) 81. (a) 82. (a)
83. (c) 84. (d) 85. (a) 86. (a) 87. (b) 88. (a) 89. (b) 90. (b) 91. (b) 92. (a) 93. (b) 94. (b) 95. (a) 96. (d)
97. (b) 98. (c) 99. (b) 100. (a) 101. (b) 102. (b) 103. (d) 104. (c) 105. (a)

Website: www.acingclat.com
YouTube: https://www.youtube.com/channel/UCCxsyMbrGVvbYu6Ua2eTb0A
69. B; Supporting lines are: “Section 3 – It provides for punishment to a person for keeping a
brothel or allowing premises to be used as a brothel or who is in charge of any such premises either
by himself or through a tenant, occupier, etc.”

70. C; Supporting lines are: “Section 4 – It provides for punishment to any person over 18 years of
age, living on the earnings of prostitution of another person.”

71. D; all the three options are not appropriate answers.

72. D; the brothel could be closed down by the magistrate if it is within 200 meters of any “public
place”.

73. D; Since A’s age is not mentioned in the question. If A would have been a major, he would be
punished under section 4.

74. C; Supporting lines are: “Section 4 – It provides for punishment to any person over 18 years of
age, living on the earnings of prostitution of another person.”

75. B; Supporting lines are: “Section 3 – It provides for punishment to a person for keeping a
brothel or allowing premises to be used as a brothel or who is in charge of any such premises either
by himself or through a tenant, occupier, etc.”

76. B; Supporting lines are: “Section 18(e) of the Indian Registration Act, 1908 specifies that the
registration of a will is optional.”

77. C; Supporting lines are: “A will or any part of the will, the making of which has been caused by
fraud, coercion, undue influence is bad in the eyes of the law.”

78. A; Supporting lines are: “It is well-settled law that once the execution of a will is proved, the
burden to prove that it was fabricated or manufactured or was obtained by committing fraud,
coercion or undue influence is upon the shoulder of the objector of such will.”

79. C; Supporting lines are: “Along with the testator, the will must contain the signature or thumb
impression of two witnesses, witnessing that the will belongs to the testator. If any of these is not
present, there is a lack of due execution in the will and the same can be challenged in the court of
law.”

80. C; Supporting lines are: “A will although registered can be challenged in the court of law. The
mere fact that a will has been registered will not, by itself, be sufficient to dispel all suspicions
regarding it.”

81. A; Supporting lines are: “Therefore, till the act remains to be a threat, it is considered as
assault.” In Stephens v. Myers, the act of showing clenched fist by the defendant towards the
plaintiff was held to be assault.

Website: www.acingclat.com
YouTube: https://www.youtube.com/channel/UCCxsyMbrGVvbYu6Ua2eTb0A
82. A; In Pursell v. Horn[vi], throwing of water was held to be assault.

83. C; Supporting lines are: “Assault becomes battery if the person apprehending the harm is
inflicted with that harm as apprehended by him.”

84. D; A will not be liable as B threw water at A.

85. A; In R. v. S. George[iv], it was held that, even if the pistol is not loaded, it may cause assault
when it is held at such a distance that, if loaded, it may cause injury. Supporting lines are: “Assault
as a tort will be completed when one person by his act creates an apprehension in the mind of the
other person that the former will be committing battery against the latter.”

86. A; B will be held liable for assault only since no actual physical harm was caused to A.

87. B; Supporting lines are: “It is thus considered that assault precedes battery, but it is not essential
that every battery should include an assault.”

88. A; X loosens the muzzle of a ferocious dog knowing that the dog will cause harm to Y. So, X has
committed an assault upon Y.

89. B; Supporting lines are: “No, if the victim was not in a position to apprehend as he was not
aware of the use of criminal force by the accused against him, then battery will not include assault.”

90. B; Supporting lines are: “No, if the victim was not in a position to apprehend as he was not
aware of the use of criminal force by the accused against him, then battery will not include assault.”

91. B; There is no consensus-ad-idem, there can be no contract. Supporting lines are: “Therefore, it
can be understood in a way that in the absence of meeting of minds, a contract so formed is void ab
initio (i.e. null and void from the very beginning).”

92. A; Since both the parties agree for 80,000..

93. B; Since there is no meeting of minds.

94. B; Since there is no meeting of minds, the agreement is void and the french jeweller is not liable
to pay.

95. A; Since A preferred X brand, which means he can adjust with any other brand too. And the
fact that he paid and took the other brand makes the agreement valid by consensus ad idem.

96. D; Supporting lines are: “The petitions should instead ask for scrapping all grounds for divorce,
across communities. ”

Website: www.acingclat.com
YouTube: https://www.youtube.com/channel/UCCxsyMbrGVvbYu6Ua2eTb0A
97. B; Supporting lines are: “I say our divorce laws are violative of Indian constitutional principles
too. They restrict a person’s fundamental right to life and personal liberty and with no reasonable
basis.”

98. C; Supporting lines are: “Unfortunately, Indian courts and the legislature have been too
conservative on divorce.”

99. B; Supporting lines are: “I can understand a legal contest over custody of children and
maintenance amounts. But contesting a divorce is like forcing one’s spouse to stay married against
their will.”

100. A; Supporting lines are: “Why should a person need to fight a battle for divorce if they do not
want to live with their spouse anymore? I can understand a legal contest over custody of children
and maintenance amounts. But contesting a divorce is like forcing one’s spouse to stay married
against their will.”

101. B; Supporting lines are: “The lender has the right over the property for the due course of the
loan period, he can put the property on rent or use it for other purposes until the repayment of the
amount.”

102. B; Supporting lines are: “if he/she is unable to pay back the borrowed amount in specified
time duration, then the lender can sell the property to anyone to get his money back”

103. D; Supporting lines are: “When the lender assures the borrower that the rate of interest will
remain the same throughout the loan period is called Fixed-Rate Mortgage”

104. C; Supporting lines are: “In this case, the lender lends money to the borrower on a monthly
basis. The entire loan amount is divided into instalments and the lender gives the borrower that
money in instalments”

105. A; Supporting lines are: “In this type of mortgage, the title deeds of the property are given to
the lender. This is a common phenomenon in banking mortgage loans. It is done to secure the
property.”

LOGICAL REASONING
106. (c) 107. (a) 108. (d) 109. (b) 110. (b) 111. (c) 112. (c) 113. (a) 114. (c) 115. (a) 116. (d) 117. (b)
118. (d) 119. (c) 120. (c) 121. (b) 122. (a) 123. (d) 124. (c) 125. (c) 126. (b) 127. (c) 128. (a) 129.
(d) 130. (c) 131. (b) 132. (c) 133. (a) 134. (c) 135. (d)

106. Ans is (c) as the crux of the passage is that even though China hints at disengagement, India
should not let down its guard as at previous occasions as well, China has backstabbed India under
the garb of pretending to disengage.

Website: www.acingclat.com
YouTube: https://www.youtube.com/channel/UCCxsyMbrGVvbYu6Ua2eTb0A
107. Ans is [a] because the first statement of the 2nd paragraph says so. [b] has not mentioned it in
the passage. [c] and [d] are not true as non has disengaged with another in the border crisis.

108. Ans is [d] as China act of disengagement was strikingly opposed to its usual strident posturing
with neighbours with whom it has boundary issues. The first and third sentence state the exact
opposite and the second one cannot be inferred.

109. Ans is [b]. This can be solved through elimination. The first is eliminated because we need to
back down at some point and not ‘never’. [c] may be correct but cannot be inferred. [d] is the exact
opposite of what the author has to say.

110. Ans is [b]. The references of Galwan valley and Doklam have been given to forewarn the
Indian troops about the usual Chinese attitude of being unreliable.

111. Ans [c] is the correct answer as can be inferred from The French president’s words. [a] is the
exact opposite of what the French think and feel. [c] is not mentioned in the passage about the
president's opposition to exchange of culture per se but he is opposed to importing dark social
science theories from America. [d] is out of context.

112. Ans is [c]. The first statement finds a mention nowhere. The second is an all French and
American university encompassing statement and hence incorrect. [d] is true but not the main
point of the passage.

113. Ans is [a]. The French are troubled, annoyed and upset at the import of American social
science theories and its implication on France and French Culture. They are not sad but have rather
angry undertones. [c] and [d] are not the meaning in the context.

114. Ans is [c] as French president mentions that the theories could divide the country by bringing
hostilities between different ethnicities. Other options are not even mentioned in the passage.

115. Ans is [a] due to the use of the word “Further” in the president’s words. [c] would have been a
more appropriate answer if the word “not” had been absent.

116. Ans is [d] as can be inferred from “crushing dissent under the jackboot”. The other options
may or may not be true but cannot be inferred from the passage.

117. Ans is [b] because that is the most logical assumption of the statement made by the author.
The government had reprimanded twitter for not taking down offensive tweets however the
government itself failed to notice the tweets of those accounts it follows because logically the
author presumes that they make offensive tweets as well. [d] is incorrect because favouring the
government is not offensive.

118. Ans is [d] as that is exactly what the author concludes as she goes to the extent of calling the
government paranoid for doing so. [a] has not been hinted anywhere in the passage. [b] is not what

Website: www.acingclat.com
YouTube: https://www.youtube.com/channel/UCCxsyMbrGVvbYu6Ua2eTb0A
the prime minister did but what the author wants him to do. [c] is not supported by any statement
as the arrested journalists could have genuine concerns as well

119. Ans is [c]. The other statements may be true but that is not what the author is trying to show
by mentioning twitter. She wants to show the extent to which the government is curbing dissent
on every platform.

120. Ans is [c] as the author states that the government is not paranoid but its acts of curbing the
genuine dissent and voices of the people is a sign of paranoia and that has consequences.

121. B; The passage discusses how history is an accidental chain of events, and how this chain
shapes our thoughts and dreams. Options A, C and D, do not trace this chain of events, or are
unrelated to history. Only option B is the best course for citizens to learn history.

122. A; The passage explains that history does not serve as an exact guide on what to do. Thus,
option A is correct. Option B is refuted by the last line. Options C and D are irrelevant.

123. D; Eclectic, by definition, means ‘deriving ideas, style, or taste from a broad and diverse range
of sources.’ The last few lines of the passage discuss that history helps us to think and dream
differently and thus gives us many options. Hence the answer is Option D.

124. C; The first paragraph discusses the author’s views that history cannot be used to accurately
predict the future. The first and second statements both disprove this view, and show that historical
events can in fact accurately predict the future. Hence, option C is correct.

125. C; The passage starts off by saying ‘If history doesn’t follow any stable rules..’, hence the
answer is option C.

126. B; In the given passage, this obligation can vary depending upon where the mediator is
practising and it may occur as a result of their employment contract or of the law. Therefore, this is
definitely false.
127. C; In the case of serious crime, breach of confidentiality could be considered legitimate. This
indicates that there is no legal obligation. Therefore, it is definitely false.
128. A; There are situations that may occur where the mediator is under obligation to break the
confidence. Therefore, it is definitely true.
129. D; As per the passage, we cannot conclude that most Mediators agree with the conditions
that the law places on them relating to breaching client confidentiality. Therefore, it is definitely
false.
130. C; As per the passage, a mediator needs to discuss with the client, whenever there is a serious
issue. Therefore, the given statement is probably false.

Website: www.acingclat.com
YouTube: https://www.youtube.com/channel/UCCxsyMbrGVvbYu6Ua2eTb0A
131. B; The last line of the passage clearly hints toward there being multiple viewpoints, despite
the majority enjoyed by the Indian National Congress. None of the other options are supported by
the passage. Hence, option B is correct.
132. C; In the first para, it is mentioned ‘The Muslim League boycotted the Constituent
Assembly's early sessions and called for assemblies to produce texts for two different countries.’ It
can be inferred that the Muslim League preferred to make their own constitution since they
envisaged a different country. Thus, option C is correct.
133. A; That the Assembly had differing viewpoints is mentioned in the last line. The second para
starts by discussing that only a quarter to a fifth of the population voted and elected the Assembly,
which can imply that it was not fully representative of the population. Hence, option A is correct.
134. C; Both the statements provide a possible reason for the contrasting viewpoints in the
Assembly. It can be assumed that adequate voting power and procedural safeguards empower the
smaller groups to be equally represented. Thus, option C is correct.
135. D; It is important to note that the question assumes all of the statements to be true. The
author’s main argument is that despite being non-representative, the Assembly had several
viewpoints and intense debates. In such a case, the last option most effectively counters the author’s
argument, making it the right option.

QUANTITATIVE TECHNIQUES
136. B; Area of room = 18 × 12 = 216 sq. feet; Area of bed = 3.5 × 6 = 21 sq. feet; Area of table = 3
× 4 = 12 sq. feet; Area of chair = 2 × 2.5 = 5 sq. feet (Here, not required to find the area of books as
it is kept on the table). Therefore, required area = 216 – (21 + 12 + 5) = 178 sq. feet
137. C; Area of table = 3 × 4 = 12 sq. feet = 12 × 144 = 1728 sq. feet [Since, 1 feet = 12 inch]
Area of all three books = 3 × 12 × 6 = 216 sq. inch; Area of lamp (circular) = πr2 = (22/7)
× 3.5 × 3.5 = 38.5 sq. inch. Therefore, required area = 1728 – (216 + 38.5) = 1473.5 sq.
inch
138. A; Required number of table = (area of floor/area of table) = (18 × 12)/(3 × 4) = 18
139. B; Volume of room = 18 × 12 × 12 × 12 × 12 × 12 cubic inches
Volume of book = 12 × 6 × 3 cubic inches;
Therefore, required number of books = (18 × 12 × 12 × 12 × 12 × 12)/( 12 × 6 × 3) =
20,736
140. C; Required number of chairs = (Area of room/Area of chair) = (18 × 12)/(2 × 2.5) = 43
141. B; In 2007, out of the total population, 30% of the population were affected by Malaria and
Out of that 30%, 10% were housewives. We know, Total Population = 1000
∴ The number of house-wives affected by malaria in the year 2007 = 10% of 30% of 1000 = 0.1 ×
0.3 × 1000 = 30
142. D; The number of housewife, students and driver in the ratio = 20:11:9
Let assume common factor = x
So, 20x + 11x + 9x = 1000 => 40x = 1000
x = 25. So, total no. of drivers = 225

Website: www.acingclat.com
YouTube: https://www.youtube.com/channel/UCCxsyMbrGVvbYu6Ua2eTb0A
Now, in the year 2009, 45% of total population affected by Malaria = 450
Out of 450 people, 30% were drivers – 30% of 450 = 135
Therefore, the no. of drivers who were not affected by malaria in the year 2009 = 225 – 135 = 90
143. A; As we found, x = 25
Then, we can find total no. of students = 25*11 = 275
In 2006, 40% of the population affected by malaria = 400
Out of 400, 60% were students = 60% of 400 = 240 students
The no. of students not affected by malaria = 35
Now, difference between the no. of students affected by malaria and the no. of students not affected
by malaria = 240 – 35 = 205
144. C; The number of housewives affected by malaria in 2005 = 10% of 30% of 1000 = 30
The number of house-wives affected by malaria in 2008 = 10% of 20% of 1000 = 20
Ratio b/w number of house-wives affected by malaria in 2005 and 2008 = 30:20 = 3:2
145. D; Total number of students = 275 (Refer to Answer of 3rd question)
The number of students affected by malaria in the year 2005 = 60% of 30% of 1000 = 180;
Therefore, The number of students not affected by malaria = 275 − 180 = 95
The number of students affected by malaria in the year 2006 = 60% of 40% of 1000 = 240;
Therefore, The number of students not affected by malaria = 275 − 240 = 35
The number of students affected by malaria in the year 2007 = 60% of 30% of 1000 = 180;
Therefore, The number of students not affected by malaria = 275 − 180 = 95
The number of students affected by malaria in the year 2008 = 60% of 20% of 1000 = 120;
Therefore, The number of students not affected by malaria = 275 − 120 = 155
The number of students affected by malaria in the year 2009 = 60% of 45% of 1000 = 270;
Therefore, The number of students not affected by malaria = 275 − 270 = 5
Thus, 2008 had the maximum number of students not affected by malaria.
146. to 150.
Departments Graduates Post – graduates Total employees
HR 585 715 1300
Marketing 312 312 624
Finance 234 702 936
IT 1,014 546 1560
Legal 468 312 780
146. D; Total no. of employees = 5200. Total no. of employees in HR, Finance and legal = 1300 +
936 + 780 = 3,016.
147. C; Postgraduates in Legal department = 312; Postgraduates in HR department = 715;
Respective Ratio = 312:715 = 24:55.
148. A; Graduates in Finance department = 234; Total no. of employees = 5200; Required
percentage = 234 * (100/5200) = 4.5.
149. D; Total no. of postgraduates in all the departments = 715 + 312 + 702 + 546 + 312 = 2,587.
150. C; We can see from the table – Total no. of employees in the IT department = 1560.

Website: www.acingclat.com
YouTube: https://www.youtube.com/channel/UCCxsyMbrGVvbYu6Ua2eTb0A
Complete Test Analysis
English Section
Excellent Score: 23+
Good Score:17+
Need to improve if the score is below 12.
Daily practice RCs (under 450 words) of previous year papers of CAT, GMAT etc. exams. You can
refer to GMAT Club as well to practice RCs.

Current Affairs including GK


Excellent Score: 30+
Good Score: 27+
Need to improve if the score is below 15.
Current Affairs in this mock is from May and June 2021. Prepare the major current affairs news
only of May and June 2021 thoroughly.

Legal Reasoning
Excellent Score: 28+
Good Score: 24+
Need to improve if the score is below 20.
Read news about the legal issues and debates. This will increase your knowledge and will enhance
your understanding of different legal terms and phrases. Other than this, practice of the right
material is the key to increase your score in the legal section. Also, you need to be careful while
referring to different sources for legal aptitude because very less material is relevant and reliable.

Logical Reasoning
Excellent Score: 23+
Good Score: 18+
Need to improve if the score is below 15. For questions of Analytical and Critical Reasoning, you
can refer to books such as MK Pandey. For comprehension based questions, the most important
thing to carry out initially is to read and improve your speed and understanding of the given text.
Once you are good with reading and comprehension, the next step will be to identify and
differentiate different arguments, assumptions, inferences, ideas, conclusions, etc from the passage.
You can practice difficult comprehension from CAT, GMAT or GRE exams to practice questions
for Comprehension based Logical reasoning questions.

Quantitative Techniques
Good score: 10+
If you are getting less than it, no need to worry just search only DI based graph questions and
caselets questions and start practicing them. But before it, clear all your concepts regarding the
topics asked (such as percentage, SI, CI, average, time and work, trains, etc.) from a book (such as
RS Agarwal) or online through YouTube. Your Mentor of Acing CLAT, must have told you about
this too. Try to follow that.

“The Best Way To Get Started Is To Quit Talking And Begin Doing.” – Walt Disney
Website: www.acingclat.com
YouTube: https://www.youtube.com/channel/UCCxsyMbrGVvbYu6Ua2eTb0A
Acing CLAT’s One Year & Two Year Courses

Website: www.acingclat.com
YouTube: https://www.youtube.com/channel/UCCxsyMbrGVvbYu6Ua2eTb0A
Acing CLAT’s Test Series & Mentorship Session

Website: www.acingclat.com
YouTube: https://www.youtube.com/channel/UCCxsyMbrGVvbYu6Ua2eTb0A

You might also like